You are on page 1of 97
INDEX a ? ' 2 t + Series 1014 [er 2. Coding and Decoding 15-25 3. Direction Sense 26-36 4. Blood Relationships 37-43 3.— Anologies : 44-53 4. Classification (odd thing out) 54-59 7. Symbols and Notations 60-61 8. Alphabet Test 62-65 9. “Number Ranking 66 -70 10. Time Sequencing 71-73 (Arrivals, Departures and Schedules) 11. Clocks 74-77 12. Calendars 78-80 * Previous ICET Questions , 81-95 Aseriesis a sequence of numbers /letters-which together satisfy a common rule. The candidate has to identify the pattern of the Series to find out the missing umber / letter in the series, For this one needs to have flexible thinking rather than to confining to hard and fast rules. Based on various types of questions appeared recent test papers, the series can be broadly categorised into three. They are 1) Number Series 2) Letter (Alphabet) series 3) Repeating series of alphabet For better understanding, some examples on number series are given here under : 318. 4)19 Answer :(2) The given series is primie number series, The next prime number Is 17.. Ex :2) 2,5, 14,17, 2 4) 39 Answer :{2) The prime numbers are written alternately A Difference, Series z Ex:3) 2,5, 8, 11, 14,47). 23 WAY TR 2) 24 3) 20 4) 18 Answer : (3) ‘The difference between the successive numbers is 3. (17 +3 = 20) Ex: 4) 45, 38, 31, 24,17 3B 1/42 ayia : 3)10 49 Answer : ( 3.) The difference between the successive numbers is 7. (7-7 = 10), i. Multiplication Serles : Ex: 5) 2, 6, 18, 54, 162, ..... » 1458 . + 1)274 2) 486. , : 3) 1236 4) 1032 Answer : (2) Anumberis multiplied by Sto. get the next nt number {162 x 3 = 486) VI Vile. _Anuller logiz . The diflerences between the successive pairs are 3. Ex: 8) 3, 12,48, 192, . 1) 768 ?) 384 Answer : (1) A number is muttisiod by 4 Division Series = a EX: 7) 720, 120, 24, saecinrtanenDy T 1) 12 2/18 ee : Answer: (4) 720/6= 120, 1205 = 24, 24/4 =6, 63 = 2, 2/2 =1 Ex, 8) 380, 180,90,45 oe yi azz a) 225 4y1s Answer : (3) 360/2 = 180, 180/2 = 90, 90/2 - 45 & 45/2 - 22.5, r? Series : : Ex: 9) 1, 4,9, 16, 25, .. 4H 1) 28 = apa Answer: (4) The series is 1°, 2”, 3", 4° Ex: 10) 0, 4, 16,36, 64, ..... ven 144 1) 1 ayaa 2)96 4)120 Anawor : (1) The serles Is 0%, 24, 4%, 62, ete, The next numbers is 107 100 rf. 7 Saries : : Ex: 11) 0,3, 8, 18,24, 35, 48, ; 1)60 ghee, & 363 4) 64 Answer (3) Theseries is 1? 1,27 - 1, 3*—1 otc. The next mt 4) 36 .. The next number is 6° = 36 ext number is (48 +15 = 63) m+ t Series : Pes 42) 2,5, 10,17, 26, 37,... », BB 1)60 2)48 3)49 Answer: (1) The series is e 1241, + Tete. The ny 1? +n Series for) nr? —n Sorlos : Ex. 13) 2,6, 12, 20, 128 Mil, iil. XV, Another logic : The series is 12,243, 9x 4,4%5, The next number is 5x6 =30 Another logic ; The series is 2°- 2, 3°— 3, 4? - 4, 5*-§, the next number Is 6° 6 = 30 Ex: 44) 0,2, 6,42, .....,.90 4) 18 2)20 3)24 4) 26 Answer :(2) 1°-1,2°-2,9°-3andsoon f Series : Ex: 15) 1,8, 27, 64, 125, 218, 0. 1) 256 2) 343 4) 400 Answer: (2) The series is 1°, 2%, 3", etc. Tha missing numbar Is 7° = 343 n+ 1 Series = Ex:.16) 2, 8, 28, 65, 126, 217,344, sssniesosnsiessens 4513. 2) 500 3)428 4) 600 Answer: (1) The series is 1° +1, 2°41, 9° + 1, etc. The missing number is H+ 1 =518 n—1 Series: EX! 17) 0,7; 28) 68, 124)...08 0, aaa | 4128 2)300 3)240 A) 215 Answer :{4) he séries is 17-1, 2°—4,5'—1 ete. The missing number is 6° a +n Series : Ex: 18) 2, 10, 90, 68, 10, 50 1) 200 2) 210 3) 222 4) 216 Ansiver : (3) The Series is 1° + 1, 2°42, 3° +3 ele. The missing number is 6 + 6 = 222 n'_n Series: Ex; 19) 0, 6, 24, 60, 120, 210, . 1) 2RO 9) 336 a) 343 4) 950 Answer :(2) The serles Is 1 — 1,2°-2, 3° 3. ete. The missing number is 7°— 7 = 336 Another logic The series is Ox 1x2,1"2x3,2x4x 4, etc. The missing number is 6x78 =336 ' Ft + nF Series : EX: 20) 2, 12, 96, 80, 150, 1) 250 2) 252 3) 276 4) 300 Anawer: (2) The serias.is 194 12, 294 22 59492 ate, T Sing NUN a eel XW ao =r? Series : Ex 124) 0,4, 18,48, 400, ceils 7760 2) 170 3) 180 : 4) 200 Answer : (3) The series is 17-1", 27-27, 9°- 9? etc. The missing number is 6° - 6’ = 252 EXERCISE -1 Choose the missing number in the fol 1 235.7,11, 1 te gy14 4716 2 356,172.80, pecuein 417 216 ay. 4ay15 3g. 15,16 1928186... i ie 2119 3)128 4) 136 4. Bee ATS). ns ccspeecens orig 132 2) 48 3) 14 4) 192 A 41,41 23, 13,11 4720 218 3) 17 416 & 48,96.25,45,. i 15 26 > a7 4) 10 a : 17 3)9 415 B 1314,17.2 1)26 2) 25 3) 22 4) 27 PBK 4)10 oa ay14 4) 16 10 STAT ieee BO 419 221 ay2d 423 Ree 1) 100 2) 110 3) 120 “ay 124 12, PPA, GIG, TG,octersctecrsnsone 1)26 13. 18. 21. 22. 23, 24. 28. 12,4417,21,, 1) 26 1,11,22,34,47,, 1)59 114.2) 4, 12cce 1).16 2) 22 7.21,8.32) 4) a 2)43 03.8,15,24) ce : 1)80 2) 32 SHAD once 112 2} 15 96,9,48,12,60,6,72........... 1)8 2)9 713.21, A357 127 2)29 1411,26. 3 qa 2)45 244, 16,8,25 We m8 2,12,30,56,90.... 1) 102 2)110 1,9,28,49,4, 18. ocescnrece A 148 2)38 AT ABB ty Ghee. ayaa a1 9,29,41 55 1,10,8,9,27,8,64,...... Se ae ee 3) 20 3)61 3) 96 3)45 3)35 a4 4 56 BF 3) 120 3) 25 3) 42 au a3 426 4) 62 4) 48 4) 48 4)a0 4j2i ayas 4) 57 0G 4) 192 4)56 448 AIG 42 GE aL RABI ctor eet netrth anal hot Ate 28. 6,9,14,21,30,41,54,, os NNN 1)6t 2)69 3) 63 4) 70 BO; By1B BBs sss sesssee BOA 1,63 2) 89 a)77 4) 81 31, 256,125 ,60,. 7 1)26 2) 28 9) 28 4)40 82, 1,10,19,28,37,46...... 1)40 2)465 3) 50 ayn 33, 8,18,23,08,.0.0.......48,69 1/36 2}38 343 445 BA 2 TAT etn ee ETRE 1)20 2)21 3)22 4)23 BB BB Biases ssttle cM BAP ar 3)9 4)8 88 BSA TIS. z 3)168 47355 87. 1)28 229 3y30 4y31 38, O61. 18 cc. 20,21 a6 27 aye 4y19 aa: Beg SE Sn eet = ele 2)6 ay Aye 40. AGAR 1188, ny 3)63 aye4 a1 O52, N35 316 aye 42. 2,6,12,20,20.... 3)44. 4)48 43. a 33 44 44, 1,1.6.4.27,9,64..0 e ne o ayia ays 416 45. 461016, B88 2)22 yea 426 46, 23,33,45,59 o ots 313 ats AT. 29,44,26,8,48 gpm 66.12, 19 70 an aie 48. 11,14,19.2 192 335 4/42 49. §,8,9,14,15,...92729,92,93 1)24 2)22 3/23 4)2a 50, 445,450,480,480,.. 1)500 3)620 4)510 ne ae a4 ae 5)3 o12 na 81 gg 10)4 11)3 12) 49/2 14)3 15)9 18)8_ 1799 48)9 19/2 20)8 213 22)1 28)4° 24)2 26)4 26) aye 281 29)2 30/1 34)3 32)4 38)2 34)3 38)4 36) B7}4 38)3 39/3. 40)4 42 42)2 433 44)4 48)3 46)4 47}4 48)3 4012 SOS ++ L / Alph rl In this type of problems a serles of the letters. of alphabet will be given which follow. pattern or a. Sequence. : To-solve these assign numbers.1 to 26 ta the lattes of the alphabet as shown below. in some casas il is useful to assign the number in the reverse order, Let us see some examples here under EXAMPLES : seg One Letter Series : Ex: 1)A, 6,6; 6,1, ; 1d 2)K at 4)M Answer :(2') The seties is (42). Le, A+2=0,C0+2=E642=6;G42=1 ‘Tho missing lotter is¥+2=k Another logic: Skip one Jeter to get the next letter After | skip J to get K> The missing latter is K Nota : "Skip" proces gavee time, ~ Ex:?)A, B,D, G, Ky. 1)P 2)N 30 4yb Answer :(1) The series is + 1, +2, +9 ec. The missing lotter is (K +5) =P Skip Process : First no letter is skipped, then 1, 2, 3 etc. letters are skipped to get the ‘Supsequent letters. Skip 4 letters to get ‘p' after ‘k’. Ex: 3)B, E,H, K, Ny... +P 2)0 a0 4)R Answer :(3) The series is + 3. The missing letter is N+ 3=Q Another Logic : Skip two letters to get the next letter. Skip Q, P after N to get G, The missing letter is Q, Reena ee eee eee x 4) B,D, G1, L,N, 4yN 20 ap 40 Answer: (4) The series |s allernately +2 and +3. The missing letterisN+3=0 Another Logic + Skip one and two letters alternately to gat the next latter, Skip two leliors ©, P altar N ta get Q. Ex:5)B,0,E,G,K, . : 3 1M 2)N 30 ayP Answer: (7) \tnumbers are assigned, the series becomes prime number series. The next prime number is 19 and the corresponding letter is M. Ex:6) AE,10,. “4 a 2yR 3)U 4ys Answer : (3) The series is a series of Vowels. Exi7}A,D 1 Pn. yu 2 ax at. ANN, Answer : (4) Ifnumbers are assigned the series becomes square series, The next number ig 5? =25 and the corresponding letler is. Ex:8)D, FH, LdL, 1M 2)N 310 - ap Answer: (3) | numbers are assigned, the series becornes composite number series. The Next composite: number Is 14 and the corresponding letter is N. Ex: 5) A, 2,8, Y, 0, X, D,.. Boos Hu 2 3) W 4X Answer: (3) The sequence consists of two series A. B, C, D, vtc., and 2, ¥, X, ete. Two Letter Series : The former letter of the series follows one logic and the later letter follows another logic. Ex; 10) AM, BN, CO, DP, EQ, . 7 1) FG 2) FA 3)GA 4)ER Answer :{2) The former letters are A, B, C, D, E, F and the later letters are M,N, O, PQ and R Ex: 11) AB, DE, GH, JK, MN, . 4) OP 2) NO appa 4) AS Answer: (3) Atter every set of letters one letter is skipped, Skip © lo get next two letters Ex: 12) AA, CE, El, GO, re qu 2)1Q ain 4)IT Answer: (1) The former latters follow a sequence of A, C, E, G, | (42 series) and the later letters are vowels A, E, |, O, U. Hance ‘I U" Shree Letter Series : This sequence consists of 3 letters in each term. The first letters of each set follow one logic, the second letters of each set follow another logic and the third lettere of each set follow some other logic. (or the same logic in all the three cases), Ex: 13) ABD, COF, EFH, GHJ, iis HK 2) NL 3) HU 4) HIK Answer : (2) The first letters tollow a sequence af A, G, D, G, | eto. The second letters follow a sequence ot B,D. H, J etc, anu the third lettors form a sequence of D, FH, J, Lete Ex :14) CKZ, DLY, EMX, FNW, in 1) GOV 2) GOU “3 @Nu 4) GNV Answer : (1) ‘The first letters form a sarios af C, D, E, F, G, otc. “The savond lellars form alseries of K, L, M,N, O ete, and the third letters form a series of Z, Y, X, W, V etc. Ex: 15) MAB, VEG, OIE, POG; 4) OPH 2) QUIT 3) ut 4) QUH Answer: (4) The frst leters form a sartas form a sories of M, N, 0, P, Qetc., the second letters form Vowels, the third letters form pnme number series (|f numbers are assigned to letters), Ex: 16) ABC, CBA, DEF, FED, GH, . 1) JKL ay IHG 3) OFE 4) UK Answer : (2) The second term is the reverse order of first term. Note ; However, one has te think in terms ot the logic involved in the given question. These are some of the frequently asked questions in different lust papers of the competitive exams. Choose the missing alphabet in the following ‘Alphabet Serics' : 1 13. 4 EXERCISE -2 aK ay BA, Ol cascade ; No 2M aP B,W,F,S, i id 3M GJ NS DA ay VFS. YL 3)P GK, 0. Du au T.,N Ku. i 4M ayn f 8, U, 0, W, 0... ay Pp 2T iay MP, LQ, KR, JS, ojo co a= IP arte 2 310 NSI, ORG, POE, OPC..... _ 1) ROA 2) RAO 3) AOR JAZ, LEX. NIV, PO 3 1) RVR ?) AUP. 3) RUA, ZUA, XOC, VIE, TE... 1) RAV 2) RAl 3) REI ABZ, BCY, CDX, DEW.. 1) FEN: 2) ENF 3) FNN KUZ, MOX, QIV, QET...... 1) SPR 2) SER * 9) SAR AB, CD, EF... a GI 3) Gd B,C, E, Hic 1K ayn ae ayy aR ayo 4x 40 4)N 4a)J 4)R 4) AVG 4) FIA 4) SIR ay HI 48. ASE,L.. 1)P 19. GJ, FR. EL, DM... a 1) DN aco 3) CN 4) CP 20. AA, BE, Cl 1) DE 2) DL 3)D0 4) 01 | 21. BO, FH:JL,NP, ES i OT 2) RT 3) RS 4) RR | <2 AG. FH, KM, PR, is | Ax 2yVZ aw 4) UW | 28. KPA, LOB, MAG, NSD)c 1) OTD 2) OTE 3) OST 4) OSE + | 1 HQ 48 ?4. GT?7, FSY, ERX, DOW. ai = 4) CRU 2) cRY ayGPU 4 4) CPV 25. EPV, FOW, GRX, HSY..... WsTy ayy ayimz 4) TO 26, ADG, GJM, MPS, 2 $ 1) SVY 2) Avy a) Suy 4) TuY 27. WTO, ONK,..cscescee en sisegncn 1) MHE ayene do, 3) NHE "4p KLE 28. GRIP, KN, Ml esesscsersssicne - id 1)0T 2yOP 3) ON 4) OL 29. PON, RGR, TSR. VUT.... a 41) YW 2) XWV 3) XWX 4) ZWV 0. ASE, BSG, G74, OTT, ete ES Es FIN a 31, Adv, E80,1 161, O25 cares 1)RI6B 2) U 4A 27368 UIA 703, YBS,........_........ W415, VERS xcs 2)XO7 ayxce axes $89, 6517; 18H13,.. = 184116 2) 72.1 15 3)52K 16 4) 72K 15 H. KS, MQ, OO, OM. ... es aK 3) KS 4)SM 1) OVX 2) OXW 3) QXV 4) OX 37. DV, FU, HT, J5, ayLP ata * a)LA 4) MQ 38 BY, OV, FS, HP JM, ... eet x yu 2) JL 3) LM ay ld 39. 83GP D9S, EK22, a2 1) FMZ 2} 8MY 3) 6MT 4) 6MY 40. BKK, DUM, FNO; HOO, seetseceo- yuvT 2) JUS 3) JUT Ay SET 41. C16E,£36G,G 641, ii 1) 10K 2) 1100 K 3) 1000 K 4) None : 42. STD, UVE, WXB, ; 1, AZY 2)YZA }) CBA 4) None 49°. TEN; PENALEN, 5. ste 1) QEN 2) HEN 3)KEN = 4) DEN 44. DUM, EOL, FIK, GEV, . aie 1) HEL 2) HOT 3) HEN 4) HAL 48. BCD, FGH, KLM, OAS, a 1) WKY 2) XYZ a) xwy 4) YZA 46. ABC, ZYX, DEF, WVU, ... F 1) RST 2) GIH 1g) GHI 4.GJH 47. L,N,0,U,. pic 4)Y QZ ain fF aX aya 48. BLV ONU, FPT, HRS, ae 1) uTQ pasts DAP. 4) arr 49. CAT, FDW,IGZ. 1) KA 3) LHD 4) LIC. BO. BEH, KNQ, TW2, soovnnnenesone : : 1 WL 2) BOF 3) ADG 4) CFI 2 2 32 41 59 Ba He Be Ms 102 1i)i 1293 49)2 14)4 1593 18}2 17}H 182 -18)3_—-20)3 P12 22)4 232 24)4 25)9 26)1 27)2 28)3 29)2 30)? 814 32)3 33)4 M1 95)3 -36)2 37/3 38)4 994 ADE 41)2 4212. 49)2 44)4 45)2 46)3 17)2 484 49/4 SO}4 ee ene ate tenet toe Pnemenemeton Wr onntetine = LICK ET EXERCISE -3, Choose the missing letters In the following letter repeating series : 1. batab-bebs 1) babs 2) baaa 3) abbb 2._ bas abaa*b “aa Ty bbb 2) abb 3) ab 3. -aba-abaa-baa ba 4) bbbb 2) abbb Sjaaea 4, a-bba-b-aab-a- bbaabb _ 1)aaaab 2) abba 3) aabbb 5. ba" ababa~ abe - ab 4) aalob 2)aaaa 3) abab 6 a-ab-bahab-b j)aaa 2) bba 3)aab 7. -abbe-a-bbec- ab-c- 1) aboabe 2) bachca Syacaane @ a-bba--b-aab-ba 1) aabea 2} babaa Sybaaab 8 a-bbe-aab- ved - bce 1) bab 2) acba 3) abba 10. abb-baa- a - bab -aba 1Yabba 2) abab 3) ccac 11, -bes-ae-aabb-ab-cc : 1) aabea 2) abaca 3) bavab 12. aD- aa-caad-¢-anb-¢ 1) bbeaa 2) bebea 3) cabac 19. 0 - abbb - cecad - ddece - bb - ba Aabeba 2) abdbe 3) abdob 14 ¢-bhb ——abbbb - abbb - 1) aabeb 2) abeob 3) abach 15. bo-d--be-- cb 1) dbodd 2) boedd 3} debetel W224 a8 44 Se Ge FD A yD 14)3 #2)4 19)8 14j2 18)3 4) babb ‘aaa ()baaa 4) aabba 4) abbb 4}baa 4) bbeabe 4) babbb 4) caba 4) aabb 4) boaca 4) cbbac 4) aboad 4) bachh 4) bades: 10)4 CODING - DECODING In. questions based on cading-dacoding, instead of writing a word clearly. a certain codes used, | Le the letters of alphabet do not stand for them selves but they represent certain other form. Hence, | they acquire an artificial or code value-following a certain set of principies / pattern. The candidate is | fequired to find out this pattem and with the help of this decode / encode a clear word. Coding and decoding tests can be classified into various calegorivs. Wa can understand some of the categories wth the help of examples EXAMPLES ; 2 Ex.) APPLE’ fe written as ‘CRRNG' int a language then how wil! ‘SILVER’ be writton in the language ? 1) VKNWTG 2) UKNXGT 3) UKMXGT 4) UKNWGT Answer = (2) RSP EP aL tsa: gre bem [oe 2 pea CR SR, eG In the above word the carresponding number ot each letter is increased! by ? fo obtain the code. So, we apply the same rule for’SILVER! then wa get "UKNXGT’ as answer. Ex:2) ma Certain code "THUNDER is written as 'VGWIFDTE. How wii ‘1QWN POUR' be written in that code ? 1) PYERNWO 2) FNYDONWO 3) FNYDANWO “4)FNYDRWNO Answer :(3) T H U N D E AR s ge? got pee gn yok pan yee po BE ASEG? We = et cee Toh Similarly = Bits pa Saws Pumsee eu. Bi $2 ee '2 at qr? ba 4 t2 ont fe ves R oN W Q@ Ex. 3) Ina ceriain code ‘STRONG’ is wrillen as 'TVUSSM’. How will BIGGER" be written in that code ? 1) CKIKIX 2) CKILIX 3) CIKLIX 4) CKIXJL Creative Learning Institute for Comprehensive Knowlodae : CLICK Answer -( 1) oe PaO) Noe urt pets yee pee pee T af u Ss 8 M Similarly > 6 i G G —E R yt 1+ Eyes pee pee Cc K J K ay oe Ex: 4) Ina certain code ‘milk’ is calied ‘black’. ‘Black’ is called ‘fish’. ‘Fish' is called ‘window’. ‘Window’ Is called ‘nose’ and ‘nose’ is called ‘teeth’. With which organ of our body we breath ? 1) fish 2} mille a) nose 4}teath Answer : (4) The actual answer is ‘nose’, but in the code ‘nose’ Is called as ‘tecth’. So the answer is Teeth’. EX: 8) ina certain code a) AB C’- means ‘well and good’ 'b) ‘BD E'- means ‘very good boys’ c) E F G' - means ‘Boys are nich’ ‘What is the code for ‘Boys’ ? i 0B 2c HE 4)F Answer :{3) We Nave to consider the statements (b) and (n). AS"E"is reprated in those two and ‘boys’ as well, the code E represents ‘boys’. Ex: 6) ‘na certain code language each r letter is shifted to f28 — 2r)” tetter tor 12 1,2, 3 ween 13; and forr = 14, 18, ... 26 the r” letter Is shifted to (2r—277" loiter, For decoding the inverse proces Is foflowed, then what is the cade for the word ‘PEACE ? BOTAL 2) ERZUR 3)ERZVA 4) RFDBA Answer = (3) Here for the first 19 letter the r" letter is (28 - 21) i.¢ (28 —2(1}= 26, then the code for A-Z 28 — 2(2) = 28 —4 =24 = X, then the cade for B = X and o.an upto m. From 14th lo 26 the © letter is (21-27) (6 2(14) - 27 = 1, then the cade for N = A and soon. The codes ate as follows ABCDEFGHIJKLMNOPQASTUVWKXYZ tlettittsdllittsittdidtidis ZXVTAPNLJHFOBACEGIKMOQSUWY Therefore the code for PEACE = ERZVR Bradtive Learaine Institute far tamnrihedcivs Kametadas : CLICK W = SAE ZCIUUKZ ____SICHIEYA ___ AY EXKEXZ EXERCISE -7 The word ACADEMY is coded as ZBZCDLX, Wrile the codes for the following words. 1) SUCCESS. 2) RESULT 3) HARD WORK... + 4) VICTORY........ The word YDAQ2 is coded as. ZEBRA, Write the relevant words for-the following codes. 1) ZCUHADG.... 2) DANZX... 8)ZOSUSTCD..., 4) OZ0DO-.... WA=Z,B=¥,C = X and so un, writy the codas for the following words, 1) MONEY. 2) BELT... 9) LARGE 4) BOOST. 5) PROFIT. 6) MANAGER... 7)COLLECT..... 8) KING... If PRODUCTION is written as 1619477815, write the codes for the following words. 1) NOTION... 2) PORT...... 3), ROOT... 4)ROUT..... Write the words for the tollowing codes- 5) 517... 6) 7615... 7) 7166..... 8) 619... Ina certain code as the word SLATE is written-as GXRYW. Write the cades for the following words, 1) STEAL... 2)STALE... 3) TAE 4) TALES... H CONSTABLE is coded as 91, what will be tha code for STABLE? 51 2)56 a) 59 4) 60 WRUN is coded as 2, wal willbe the code for SUN? Wi 22 aa aye HFACE is coded as 15, whal will be the cude forthe word RACE? 126 227s - 3) 28 4) 30 HCAN is coded as 4215, whal will be the code for STABLE? 1) 192012125 2y202129125 3) 181712125 4) None if the word HONEST is coded as FMLCOR, liaw should the word TRUST be coded ? 4) USVTU 2) TSURT 3) RPSOR. 4) RPSOA It the word FINANCE is coded as ECNANIF. how the word RAILWAY be coded ? )YANLIRA, 2)YAWLIAR 3) SBUMKBZ 4) SBJXMB it the word SCHOOL is coded as VPKARO then how is the word COLLEGE be coded ? 1) FROOJHA 2) EGELLOC 3) EGNNGIG 4) FROOHJH if the word APRIL is coded as CSVNA, how the word AUGUST be-caded ? Creative Learning Institute for Comprehensive Knowledge: GLICK <” 14. if the word INTEREST is coded as MPtXIVIWX how the word PUNJAB be coded ? 1)SKOMDE 2)TYNEF 3) SXODME 4) TYRNEF 15. If WOMAN is coded a3 124: and SERVANT as 6789450 how can VOTERS be coded 7 1) 972086 2) 920785 9).902786 4) 978206 16. If BOMBAY Is coded as 476479 and MELUR as 63852 how can BELLARY be coded 7 1) 2376184 2) 2586179 3) 4377168 4) 4988129 17, lf the word JANAKI is coded as 725291 and KRISHNA is coded as 9316852 how can SARIKA be coded ? 1321629 2)612032 39623192 4)874639 8. . If BROWN is eotied as 78599 and GARDEN as 218643 how can ORANGE he coded 7 1)5B1324 2) 541324 3) 671342 4) 874639 19. It WALTAIR is coded as 2347305 and NELLORE as 8944759 howcan TENALI be coded 7 1) 84360 2) 798346 3) 788046 _ 4) 874639 20. If DECEIVE is written as EGFINBL, DEFENSE can be written as 1) FGHGPYT 2) EGIISYL 3 EFGFOTE 4) HUIRWI 21. The word JUDICIAL is written as LADIGIUY how is GLORIOUS written in that cade? 1). GEOIROUS 2) USORIOGL “+ 3) GUOIROLG! 4) SUORIOLG 22. The word FAVOUR is written as EBUPTS how is DAGGER written in that code ? 1) CBFHDS 2) EBMHES } 3) CBFFDS: 4) CBFHDO 28. DISPEL: IDPSLE = EFEECT socsnnnonnns 1) FEEETC 2) EFEETG 3) EETFTC 4) None 24, HUNTER : UHNTRE = MANAGE se 1) AMNEAG ?) ANMAEG 3) AMNALG 4) ANNAEG 25. RAMAYANA ; AMARANAY = TULSIDAS 4) LUSTAST 2) ULSTDAST = 3) ULTSDAST 4) None 26. CANCE: FRAG = MUSIC :., ae 1) SZWEL 2) SWZLE 3) SZWLE 4) WZSLE 27. TABLE :GZYON = JUIGE : is a 1) OFRXV 2) FORXV 3) OFXRV 4) OFSXV 28, FALEND : HUMJTK = CANDLE : +) E0SIAL 2) EDRILR 3) EERIAL EDRIAL SIE macs: cathe bv-Dainrchaedioe Meewaiies CLIO. ay. a7. YELLOW = XFKMNX = GOUNTRY «.. 1 naan 0 1) BTPOSSX 2) BPTOSSX 3) BPTOSRX 4) None . QUIGK : PSFYF = NEST x 1) MNCP 2) NCPP 3) MCPP 4) NPP. KNIFE : IFEKN = DOGTOR eeepc Sn licn 1) CTORDO 2) COTRDO —-3), CTODRO 4) None PEOPLE : PLPOEE = TREND :. vacooventiiten hs amcaseanesie! 1) TRAEN 2) TREND 3) TNERD 4) TRNDE FOUGHT : EQUKGZ = MALE -. ani ns 4) LUG 2yLICl 3) LCI 4) LCCl HUMIDITY - UHMIIDTY = POLITICS =e 1) OPLTICS 2)OPLITCS 3) OPUITGS © 4) None . CALANDER ; CLANAEDR = CIRCULAR: za Raa 1) GRUIGALRL 2) CIRGUALR =) G@RIUGAL CAICALR GOODNESS ; HNPCODTR - GREATNESS 333 1) HOFZUMPRT 2) HQZFUMFAT, 3) HRZFUMFRT 4) HQFZMUFRT * ROBUST : ONATRS = .. 2: GBNPVT 1) ANGER 2) BANAN 3) APPLE 4) ANARK SWITCH :TWJSDG = ..... ea s COFZE 1) BERDA. 2) BREAD ~ 4) BARED REMOTE : ROTEME mas 3s PNIICC: 4) PIINIC 2) PCINIC 3) PGGNIG: 4) PICNIC | CRICKET = FULFNHW a . EULGH 1) BREAK 2) BRIDE 3) BROOM 4) BRAKE CURTAIN : GAITURN = HILLOGK :. = 1) HOCIUK 2)HOCLILO 8} HOCLILK 4) None FASHION : FOIHSAN = PROBLEM 1) PELBORN- 2) PELBORM BLEMISH | ADDPHWG = CHAPTER 2 oe 4) BRZSSHA 2) BKSSZHO Creative Learning Institute for Comprohensive Knowledge : CLICK a7, 48 49. MENSION : NEISNOP = FOLIAGE «... 1) EOAILGF 2) EOAILGE 3) EAOLIGF 4) None GASLIAL > GEWYEP = PEOPLE «.. e 1} ITSTPL 2) ITSTPI 3) TISTIP 4) TISTP! SALE: PAIE= CASUAL 20.0000. ae 05s 1), AZPUXL 2) ZAPUXL 3) ZPAUXL 4) ZPAXUL, CENTRAL * DIOUSDM + GIRGT 4) DISDMFT 2)DSODIMT —_3) DOSDMIT 4} None NADIR: OFEOS = STEAMER :....._ Sao a ae 1) TUIENIS. 2) TUEINIS 9) TUEINIS 4) Nana MISCHIEF : NOTDIOIG = UTENSIL - 1) AUDIOTO 2) AIUOTOM, 4).None ELEPHANT : SMZGODKD = ZEBRA 2 eee 1) ZOADY - 2) YDAQZ 3) ZAQOY 4) None ‘KEY +) 1)HTEBURA Z)QORTKS 3) GZOCVNGJ 4) UHBSNOX 2) 1) ADVISER 2) ESSAY 3} APTITUDE 4) PAPER 3).1)NZM 2\yZMP 3)OZYLFL 4yYLB 5) KILURG G6) N7M7 TV TyAZUS 8) XOVIP: 4)1)S17B15 23167 sper? Ay6IAT a) NOT 5) TRON 7)TORN a)ROD BVGYWRE 2)GYRXW ayyRW ay YRS 83 73 82 92 104 1)? 12)4 19) 14)4° 15/2 16)4 1773 18)1 1992 20)2 24 22)1 B81 2aS s)2 26)3 271 284 29)2 . Ws 31}1 3233 33)9. 34}2 35}4 36)1 37/3 982 3H4 sme 44)2 4292 43)3 44)1 45)4 46)2 47)3 48)t AS SHY ++ ‘Creative Learnina Inctitute tor Comprehensive Knowiedge : CLICK a 0. 1a EXERCISE -2 DEAF: 16=LEAF:.. at 1)25 2)36 3)24 4) 50 DAD-214=SAD: te Pee nee 1) 1961 2)1914 3) 1941 4) 1937 POLICE: 40 = CONSTABLES 4130 2) 40 4)60 KALAJAM- 7= BARFI 5 26 aja RAMESH :4 = SURYENDRA ; 4)6 yt 46 STOP :8-SP-OT:. ya 21 ape. 43 MOON 5-2 = STAR 5 nnsnseine 4)-2 2-4 “Bes 4-5 MAN: 28=RAN:. : a 4)28 239 She ae 343 a)ss EYE:46=EAR |. fi 4) 53 2) 60, 357 4)59 Directions (11 - 20) : in cortain code language each r jetteris shifted to (27 =r)" latter of the alphabelie. A Z,B 3 ¥,C > X 182 a AY aA Ae Ze Which word is coded as "XLYI2Z" 2 _.Z.-3 A. For decoding an inverse process is thilnwed 1) COBRA 2) CRACK 3) CREAT 4) CARGO Which word is coded as H70G O7PV"? 1) SAIL LORE 2) SONGTRUN 3) SALT LAKE 4) SALT LKAE ‘What is the ode word for WATCH' 7 1ozexs 2) DZHVS 3) D7evS 4) BZAKE 14. What is the cade word tor ‘INDIA’? 4) RMWZR 2) RMWRR 3) RMWPZ 4) AWM 15, What io the code word for LABOUR"? 1) OR¥rU 2)0ZVMF 3) OYZLFI 4) OZYLFI | 16. Which word is coded as 'NZIXSV' ? 1) MARKEL 2)MARKET —) MAFIGHE 4) MIRCHI 77. What is the code word for ‘SFAVICE ? 1) HVIERXY 2) HVJERXV 3) HUIERXV 4) HVRERXY 18. What is the code word for ‘MARCH - FAST’ ? 1) NZIXSVZHG 2) NZIXSUZHG (8) NZIXSUHZG 4) NZIXUSHZG 18. Which word is coded as 'HVOLLM 7 1) SELOON - 2) SELDOM 3) SELLCM 4} SALOON 20. Which word is coded as ‘KVZXLKO'? - . 1) PEOPLET 2) PEACOCK 3) PEACOGI 4) PENTAGN . Directions (21 = 30): tn a certain code language wach" leiteris shifted to (27 — 2r)" letter tor] 11,2.) coun, 18; the 14! loner is shittad to-26 letter and for r = 15, 16, 26 the c letter is shifted to (2r— 28)" letter. For decoding the inverse process is followed. 21. Whatis the code forthe word ‘PONY? 1) BOZV 2) QPOZ 3) DBZV 4) DBZU 22. What is the onde tor 'WKMT'? : 1} RUBE 2)REAL 3) REEL 4) ROAL 23. Which letter Is coded ag 1! 2 1yM 2K 3) 48 PA, What is the code for'GKWLA' ? 1) MERCY 2) MERIT 3) MERCL 4) MERLO 25. Which is the code for T? no 2M 38 aL 26, Which word Is coded as INTER’ ? 4) ZILOH 2) (ZLOH 8) 1710 4) we 27, Whatis the code for ‘PKYFT' ? 1) DEVoo 2) TEMPO 3) DEPOT 4) DRUGE 28. What is the cade tor 'NKMT! 7 . ek RE SN Creative Learning Institute tor Comprehensive Knowledge: CLICK 3. ‘Which word is coded as ‘VERY GOOD’? 1) POHVMBES: 2) PQHUMBES = 3) POIYMBBS 4) PQJV MBBS ). Which word is coded as 1GET' ? 4) 1UQM 2) QL 3) VOU 4) NON If 1. SKI APS TRI stands for "NICE SUNDAY MORNING" 2. THE STI RPS stands for “EVERY TUESDAY MORNING" 3. SKI PTR QLM stands for "NICE MARKET PLACE" what Is the code for SUNDAY? 1) SKI 2) STI 3) TRI 4) RPS . if 1. KA BI PU YA means “YOU ARE VERY INTELLIGENT" 2. Ya LO KA WO means * THEY SEEM VERY INTELLIGENT” 3. LA PU LE means " YOU CAN SEE" 4. SUN PUN YUN YA means "HOW INTELLIGENT SHE 1S" what Is the code for "ARE"? 1) KA 2)BI 3) PU 4) YA 11, GHIKAI SHI means “EARTH IS ROUND’ 2, CHU CHIN CHI means “BANANA IS SWEET" 3, KUL SHAK KAI means "BALLS ARE ROUND” what is the code for "EARTH"? & 4) KAI 2) CHL 3) SHAK 4) SHI If 4. NEE TIM SEE means "HOW ARE YOU" 2. BLE NEE SEE means "WHERE ARE YOU" what is the code for "WHERE"? 1) BLE 2) NEE 3) SEE 4)TIM 11, MUM SUM DUM means "WATER 1S PURE" 2. LAO CHAIN MUM means’ "WATEH HAS TASTE" 3. DUM PHU PHIN means "HE IS HONEST" 4, KHU SING CHAIN moans "NEVER TASTE IT whatis the code tor "HAS"? 1} LOA 2) CHAIN 3) MUN 4) None Which statement in Q. 35 Is Superfiuous? yi 2)2 3)3 4) None Creative Learning Institute for Comprehensive Knowleage : CLICK AO, al. If 1. PIT NAE TOM means "APPLE IS GREEN" 2. NAE HO TAP means "GREEN AND WHITE” 3, HO TOM KA means “SHIFT 1S WHITE" what is the code for *APPLE"? 4) PIT 2) NAE 3) TOM 4) None 3. If. 1. RUM PUM PO ingans "BOY IS MAD" 2, MAIN PO SHE means ‘GIRLIS BEAUTIFUL" 3. DUM PUM TO means "THEY ARE MAD" what is the code far "BOY"? 1) PO 2) RUM 3) PUM 4) DUM . If 1. MX¥ DAS ZCI means “GOOD LITTLE FROCK" 2. JX COS ZC! means "GIRL BEHAVES GOOD" 3. HVG DRS COS means "GIRL MAKES MISCHIEF" 4. DAS AUP GOS means "LITTLE GIRL FELL" what is the code for "FROCK"? 1) DAS. 2) MXY 3) ZC1. * 4) cos if 1, KEMP LUMP ‘TEMS moane “SPEAK THE TRUTH’ 2. BIS TIM NAK means "ALWAYS ‘SEEK | oer 4. 1IM TEMS SIK means "KNOWLEDGE ISTRUT! ' 4.LIK BIS ZAP means "NEVER SEEK VOILENCE* whalis the Gude for "ALWAYS"? NBIS 2) LUMP 3)TIM 4) NAK f SKY is called LAND, LAND is called CLOUD, GLOUD is called RIVER, RIVER is called OCEAN, and OGPAN is called FART, then where do WHALES live? 1) Ocean 2) Land 3) Earth 4) River . Hf FRUIT is called APPLE, APPLE is called ORANGE, ORANGE is called PINEAPPLE, PINEAPPLE is called MANGO and MANGO is callad BANANA, than what keeps the doctor away, when we eat it daily? 1) Apple 2) Pineapple 3) Orange 4) Mango If ANDHRA is called TAMILNADU. TAMILNADU is called KERALA. KERALA is called MAHARASHTRA, MAHARASTRA Ip callod MADHYA PRADESH, and MADHYA PRADESH is valled ORISSA, then where does MALAYAL| speaking people live? 1) Andhra 2) Maharastra 3) Kerala A) Orissa Creative Learning Institute far Camprehensive Knowledge : GLICK 47. |. If COPPER is called SILVER, SILVER is called GOLD, GOLD is. called DIAMOND, DIAMOND Is called RUBY and RUBY is called EMERALD, then what is KOHINCOR? 1) Diamond 2) Silver 3} Ruby 4) Emerald |. If HEAD is called EYE, EYE Is called MOUTH, MOUTH is called HANDS, HANDS are called LEGS, LEGS are called EAR and EAR is called TOUNGE, then with which of following would a person TASTE? 1) Head 2) Mouth 3) Hands 4) None. . If BALL is called BAT, BAT Is called WICKET, WICKET is called STUMPS, STUMPS are called EXTRAS, EXTRAS are called UMPIRE and UMPIRE is called CRICKET, then, what were there on the WICKETS? 1) Ball 2) Bat 3) Umpire A) Extras If PLANET Is called MARS, MARS is called VEENUS, VEENUS is called EARTH, EARTH is called MOON and MOON is:called SUN, then which is the beautiful planet? 1) Earth 2) Mars 3) Sun 4)Moon if RUBBER Is callod PAPER, PAPER Is called CHAIR, CHAIR is callad CLOTH, CLOTH is called GRASS and GRASS Is called WHITE, then with which item the chappals is made with 2 ; 1) Gloth 2) Grass * 8) Paper 4) Chair IfKING is called PRINCIPAL, PRINCIPAL Is called QUEEN. QUEEN is.called TEACHER, TEAGHER is called WATCHMAN and WATCHMAN is called SOLDIER, then who is the head of the institution? 1) Teacher 2)King =~ — 3) Watchman 4) Queen lf GUITAR is called KEYBOARD, KEYBOARD Is called JAZZ, JAZZ is called TABLA, TABLA is called FLUTE, FLUTE is called HARMONY and HARMONY is calied MOUTHORGAN, then on what instrument Mr. Ustad ZAKEER HUSSAIN is a specialist? 1) Jazz 2) Flute 3)Mouthorgan = 4) Harmony 8 22 98 42 58 G4 71 H2 H2 8 41)4 42)9 1S}4 44)9 154 16)3 ATH 18/2 HT 208 213 222 23p4 24)1 25/4 26)4 271 28)1 29)1 30/2 31)3 -32}2 33)4 -34)1 35}1 -36)3 B7}1 _3aj2. 39/2 ANA 41} 42) 45)2 44)9 45)4 46)4 47)1 48)3 40)4 50)2 Creative | earning Instittle far Ramnreheasive Knowindas : CLICK in this chanter the ‘questions consist of a sort of direction puzzle. A successive follow up of directions is-followed and me candidate 's required to ascertain the final direction or the distance | between two points L.e. starting and ending point. Noth East West South am7 iiwe EXAMPLE : { Aller travelling 70 km, | turned right and travelled 6 km, then tumed right and covered ; a distance of 3 km, now | am mowing towards East, In which direction did | start my journey ? ae 1) Nocth 2) South 8) East 4) West Answer : (4) bg 3km. $ . sia ftokm ‘ (Sp = Starting Point) Se, al thet beyinining | weril lowards west, 1 Grnative Learnina Instituts far Qamnrchensisa Kiaradodan CLICK EXERCISE -1 4. Your house is facing east side. Your friend is coming to your house. What is the diructivn (o left side? 1) South 2) North 3) East 4) Wost 2 {yo 10 km towards North from my house then | turn left and go 20 km, then again turning left | o> 10 km. in which direction am | now fram my house? 1) North 2) South 3) East 4) West 3. Directions :A,B,C,Q.E & F are six Villages. Curelully study the following information and answer the questions, Ais 2 km west to B.F ia 3. km soutnto D.C Is tkm south lo B, Dis 3km east to 6 Dis 7 km west to E, f Which three villages areinva lime... 1) ABG 2) CDE ‘S) EFC ‘ 4) GAF DIRECTIONS - (G.No. (4) ta (B)) : These questions are based on the following informat on: Vinay starts from his house along with his wita at 9.90 A M. inthis car. He goes 1 km east, crops his wife at her office, tums left, goes aeer km. Then turns right, aftor a km, reaches th. Gank. Where he spends tive minutes, then he tine towards north and reaches hospital which is 1 km from the bank. After talking to a doctor for five minutes he tums towards west and reaches tis office, which is 2 km from the hospital at 9.58 A.M. 4. How far is Vinay's office from his home as the crow fes? 4)3km 24%, 3)2km 4) 5 lem. 5. How faris hospital from his wifa's office? 4) 3km 2) 4km 34 2km 4) tk & How much time would Vinay have teke in reaching his office by the same rovle, if he had not stopped atthe bani and hospital. 4}.25 mins. 2) 20 mine. 3.23mins. 4) 1smins, 7. What is the average speed? 1) 15:kmph 2) 18 kmph 3) 25 «mph 4) 20 krrmahh & At what time did Vinay reach the bank? : 1) 942M. 2)999AM 3) 945 A.M. 4/936 A.M Paneth t anenine Inetitite for Comacchensive Krawledae t cuick © 9 Vivek starts walking towards North, After walking 25 km, he turns left and walks a distance of 10km, then he tumed right and walked 5 kim, and again he turned to his tight walked 10 km, and finally he turned to his tefl and walked 18 km. What is the straight distance between the starting and ending point? 1).35 km, 2) 56 km 7) 50 km, 4) 46 km 10. Praveen starts walking straight towards east. Alter walking 75 meters, he turns to the left and walks 26 meters straight. Again he turns ta the eff, walks a distance of 40 meters Straight, again he turns to the left and walks a distance of 25 meters, How far Is he from the starting point? 1) 140 Meters 2) 50 Meters 3) 115 Motors 4) 95 Meters 11. Man! walks 10 km. towards North. From thero, he walks 6 km. towards South. Then, he walks km. towards East. How far and In which direction is he with reference to his starting point ? 1) 7 km, East 2)5 km, West 3)5 km. North-East 4) 7 km. Wast 12. Ajay leaves for hls Office from his house. He walks: towards east. After moving a distance ot 20m. he tums towards south and walks 10m. Then he walks 36m towards wost and {urther Sm towards north. He then turns towards eas! and walks: 15m, Whatis the straight distance in meters «between his initial and final position? 10 26 F 18: 4) None 13. Mr. Rao walks 20 meters towards Noril. He then turns left and walks 40 meters. He again turns left and walks 20 meters, Furthar, he mavas 2( meters after turning to the right. How far ls he how frorm his original position? 120m ?)30m #5 3) 80in 4) 60m 14, Ramesh startert waiking towards North. Alter walking 30 meters, he turned left and walked 40 meters. He then turned lell and walked 30 meters. He again tumed lett and walked 50 meters. How far was he from his original position? 1) 50Mts. 2) 10 Mts, 3) 40 Mts. 4) 20 Mts. +5, Kumar starts from his house towards West, Atter walking a distance of 30 mis, he turned towards right and walked ZOmts. He then tumed eft and walked a distance of 10 mts, turned to his lott Again and walked 40 mts. He now tums to the lefl and walks Smits. Finally, he turns to his left. in which direction is he walking now? 1) North 2) South 3) East 4) South-West 16. Krishna leaves from his Home. He first walks dOmis in North-West direction and then 30m ir South-West direction. Next he walks 30 mts'in South-Fast direction. Finally, he turns towards his house, In which direction is he moving? _ 2) Northwest 3) South-East 4) South-West 7. 19. 21. ). Triveni starts walkirig towards Eas. Aller moving a distance:of 20 mts, he tums to his left and Srinivas walks 10 mts towards the South. Turing ts the lett, he walks 20 mts and then maves to his right. After movirig a distance of 20 mits, again he tums to the right and walks 20 mts. Finally, Ine turns to the right and moves a distance of 10 mis. How far ard in which direction is he from the starting point? 1) 10 mts North 2) 20 mts South 3) 20 mts North 4) 10 mts South Priya goes 30 mts North, then she tums right and walks 40 mts, again she turns right arid walks | 20 mts. She again turns right and walks 40 mts. How many meters Is sho from her criginat position’? 10 2) 10 3)20 4740 Ranga walks 90 rts towards South. Then turning to his right, he walks 40 mts. Then turning to his left, he walks 20 mis. Again, he turns to his left and walks 30 mts. bow far is: he trom his init position? 1) 30ints 2) 20mis 3) 50mts! 4) 60mts walks 15 mts. Then he tums to his right and movasi25 mits. Next, he turns to:his right again and ‘walks 16 mts. How far ls she from her starting point? 1) 0m 2).25m 3) 40m 4) 45m Starting from a point’P", vijay walked 20m towards South. He wrned Ieft and watkad 30 m.He then tumed teft and walked 2f1m. He again tumed left and walked 40 m and reached a point Q. How farand in which direction is the paint Q from the point P? 1) 20m West 2) 10m East - 3) 10m West 4) 10m North Ravi walks Northwards. After 6 while, he turns to his night and a ite further to his lott. Finally aller walking a distance of oné K.M. he tums to his left again. In which direction is he mowng now? 7) Nerth 2) South 3) East 4) West Badri started walking towards East. After moving a distance of 1 km, ho tumed South wards. and walked 5 km. Again he turned to East and walked 2 km, Finally, he tured to the North and walked 9 km, How far is he from his starting point? 17 km 2)5km 34m 4) 3k Facing towards the south. Maruthi started walking and turned left after walking 30m. He then walked 25m, tied left and moved a distance of tim. How far is he from his starting position snd in which direction? 1) At the starting paint 2) 25m West 3) 25m East 4) 30m East at. |. Venu travelled from a point X straighttoY at a distance of 80m. He turned right and walked Som ‘Then again tumed right and walked 70 mis. Finally he tumed right and walked So m. How taris he from his starting point? 1) 20 mts 2) 50 mts 3): 70 mts 4} 10 mis |. Siva wont 15 kms to the west from his house, then turned lefl and walked 20 kms. He then tumed east and walked 25 lens and finally lumed left covered 20 kms. How far was he from his house? 4) 5 kms: ; 2) 10 kms 3) 40 kms 4) 80 kms . Aruna travelled 7 km towards east, then turned leff and travelled 5 km. Next she tumed teft and \ravelled 7 km. How far was she from the starting point? 4) 19 km 2) 12km 3)7km A) 5 km .. From his house, Priyatha went 15 keris to the north. Then she turned wost and covered 10 kms. Then, she tumed south and covered 5 kms, Finally, turning to east, she covered’10 kms. In which direction is she from her house? 1} East 2) West 3) North 4) South Walking 50m to the south of her house, Ramya tums fefl and goes another 20m. Then turning to the North, she goes 30m and then starts walking to her house. in which direction is she walking now? 4) North-West 2) North ~ 4) South-East 4) East Laya, who is facing South turns to her left and walks 16m, then she tunis to her loft and walks 7m, then she walks 15m towards west. How far is she frorn her original position? 1)-22 mm 2)37m £ 3)44m “47m Govind walks 20 m towards North. Then he turns right and walks 30m. Now te turris right and walks 25m. Now turningleft, he walks 15m.Again, he turns left and moves 1%m. Finally, turning to {eft he again walks 18 mts. In which direction and haw faris he from his original position? 1) 1SmEast 2) 30m East 3} 16m West 4) 45m West . Subhash was going home from his school. He first moved 9 km towards south and then lured to hig left moving 2 km, from there. He moved to his left and walked 3 km. Further, he turned to nis left again and moved 1 km, to reach his house. In'wnieh direction was his house from his school? 1) North 2) South 3) East 4) West The door of Mr. Syam's house faces east. From the hackside of the house, he walks straight 50m. Thes turns te the right and walks 50m again. Finally he turns towards left and stops after walking 25m. In which direction is Syam from the starting point? 1) South-East 2) Notth-East 3) South-West 4) North-West 34. Rakesh walked 2 km West of his house and then-turned South covering 4 km. Then, he moved 3 km towards Exst and Finally he moved 1 km tawards west, How faris he from his initial position? i)2 km 24k 3)9km 4) 10 km 35. Venkatwalks 6m to the East and then turns ‘o the Soutn and walks 5 km. Again he turns tothe East and walks 6 km. Next; he tums to'North and walks 10 km How-tar is he now fram his starting point? 15 km 2) 12km 3) 13 in ay 1t km 38. Eswiar faces towards North. He walks 25 mts. atter turning to his night. He then turns to his left and walks 30 mis, Next, he moves 25 mis to his right. He then tums to his right again anel walks ‘55 mts. Finally, he turns to the right end moves 40 mts. In which dircction is he now from his. starting point? 1) South-West 2) South 3) North-West 4) South-East 37. Laxmi moved a distance of 75 mts towards the North. She then tumed to the lett and walked tor about 25 mis. Turned left again and walked 80 mts. Finally, she turned to the right at an angle of 45! In which direction was'she mavingtinally? 4) North-East 2) North-West 3) South-West 4) South-East 38. Vikram left for his affice in his car. He drove 15 sem towarels North and then 10 km towards West. . He then tuned to the South and covered Stun. Further, he turned to the East and moved & km. Finally, he tumed right and drove 10 km. How far and In which direction is he from his starting point? z 1)2 km West 2) 8 kn Bast 3) 3. km North 4) 6 kan South 39. Raghava Rao's school Bus js facing North when itreaches his sclivol. Afler slavting from Raghav Rao's house, it turns right twice and ther lurned left before reaching the schoo!_ What rlirection was the bus facing when it left the bus stop in front of Raghava Fia's houso? 1) South 2) North * 9) East 4) West 40. Veena wants to goto the University. She starts frorher home which is inthe East and comes ta a crossing, The road (oher left ends ina Theatre, straight ahead is the Hopital. in which ditesion is the University situated ? 1) East 2) West 3) North 4) South 41. Shallesh and Venkat start from a fixed point, Shailesh moves 9 km northward turns right ang then covers 4 km. Venkat moves 5 km towards West tums nght and walks 3 km. Whatis the distance between Shailesh and Venkat ? 4) 10m 2)9km 3)8 km 4) 6km Craative Laarning Institute for Comprehensive Knowledge 1 CLICK 42. 43. 45. 47. Facing the east, Ram tured left and walked 10 mts. Thon tumed to his left again and walked 110 mts. He then tumed 45° towards his right and went straight to cover 26 mts. In which direction is he from his starting pet? 1) South-West 2} South-East: 3) North-West 4) North-East A and 6 start moving towards cach ether from two places 200m apart. Atter walking GO km. B turns left and goes 20m, [her he tums right and goes 10 m. He then tums right again and comes back to the road on which he had started walking. If A and B walk ville thee same. speed, what is the distance between them now? 1)60m 2)40m 3) 30m 4) 20m I start from my home and went 2 ken straight. Then, | turn towards my right and went 7 km. I turn again towards my right and want 1 km. If | am/North-West from my house, then in which direction ‘did | went in the begining? , 1) North 2) South 3) East 4) West. A post man was returning to the post office which was infront of him to the north. When the post office was 100 mts away trom him, he turned tothe left and moved 50 mis lodeliver the last letter at Shakthi Villa, He then moved in the same direcfian for 40 mts. Then he turned to his fight and moved 100 nits. How many mts was he away from the post office? no 2) 80 3) 150 4100 3. The post office is to the east at the schaol while my house is to the South of the school. The market is to the North of the pestoffice, Ifthe distance of the market from the post office is equal lo the distance of my house from the school, in which direction is the market with respect to my schoal? 1) North 2) East 3) North-East 4) South-West The time on the clock is 6.90 pn. If the minutes hand is pointing towards weet. than the direction of the hours hand point towards ? 1) West 2) North-West 3) South-West 4) North Vikram, who is facing South, tuims 135° in clockwise and then 180° in anticlock wise direction. In which direction is he facing now ? *% 1) South 2) South Wost 3) South-East 4) East Creative Learning Institute for Comprehensive Knowledge : GLICK 49. 50. Tarun is 40 m South-West of Kiran, Vijay is 40 m South - East of Kiran, Then Vijay is in which direction from Tarun ? 1) East 2) West 3) North 4) South Kesav walks 50 km towards East, then he turns to his right and walks a distance of 15 km; again he turns to his right and walks 70 km. Finally he tums to his right and walks a distance of 30 km. In which direction and how many kms. from starting point ? 1) 30 kms. North-West 3 2) 25 kms. North-West 3) 45 km North - West 4) None 13 -2)4 «32 4B 5)T 64 7)4B)2 410) 4 11)3 12)2 13/4 14)2 15)1 16)2 17)2 18)2 19)3 20)4 21)3 22)4 23)2 24)3 25)4 26)2 27)4 28)3 29)1 30/4 31)2 32)8 -33)4 34)2~ 35)3 36)4 37)2° 38)1 39)4 40)3 41)2 42)8 43)2 44)4 45)2 46)3. 47)2 48)3 49)1 50/2 Poe re | SEATING ARRANGEMENTS EXERCISE -2 1. Ofthe Six members of a group ig in a row, Ais to the lett of D, but on to the right of B. Cis on to the right of D, but is on to the ‘oft of E whois to the left of F Which two members are sitting right in the middle? 1) Aandc 2)CandB 2)DandB 4) DandC 2. A,B,C and D are playing cards. A and B are partners. D faces towards North. If A faces towards west, then who faces towards South? yc 2)B 3)D 4) Data in adequate 3. _ Five persons were playing cards sitting round a table. Arun was to the left of Mahesh, Raju was to the right of Sudha and between Sudha and Janaki. Who was to the right of Janaki ? 1) Raju 2) Mahesh 3) Janaki 4) Arun 4. _ There are 5 towns viz. A, B, C, D and E ina district. The town A is to the west of E.Cis to the east of A but to the west of E. Dis to the east of B but to the west of C&A. Which town is the farthest west? NE 2)D 3)A 4)B 5. Awatch reads 3.30. If the minutes hand points towards East,in which direction does the hour hand pointout ? é 4) North 2) North-West 3) South-East 4) North-East 6. Six persons A,B,C,D,E and F are standing in a circular manner. B is between D and C. Ais between E and C. Fis to the right of D: Who is between A and F? 1B 2c 3)D 4E 7. Five boys are standing in a row facing East. Vamsi is to the left of Sanjay, Tilak and Srinu. Sajay,| Tilak and Srinu are to the left of Sushil. Srinu is between Sanjay and Tilak. If Tilak is the fourth from the left, how far is Sanjay from the right? 1) Fifth 2) Fourth 3) Third 4) Second 8. If Ais to the South of B and C is to the East of B, in what direction is A with respect to C? 1) North-East 2) North-West 3) South-East 4) South-West 9. Six persons A,B,C,D,E and F are standing in a circle. B is between F and C; A is between E and D; FIs to the left of D. Who is between A and F? 1B 2c 3)D 4E 10. There are four towns P.Q,R and T. Q is to the South-West of P, R is to the east of Q and South- East of P, and Tis to the north of R in live with QP. In which direction of P is T located? 1) South-East 2) North 3) North-East 4) East — sii |. Five students A,B,C,D and E are sitting in a row. Dis on the right of E. B is on the left of E, But is on the right of A. Dis on the left of C. Who is sitting on the Extreme left? 1A 2)B 3)D 4E 2. Two ladies and two men playing cards are seated at North, East, South and West of a table. No lady is facing East, persons sitting opposite to each other are not of the same sex. One man is facing South. What are the directions opposite to the two ladies 7 1) East and West 2) South and East 3) North and East 4) North and West 3. One morning after sunrise Abhi and Rahul were standing in a lawn with their backs towards each other. Abhi's shadow fell exactly towards his left hand side. Which direction was Rahul facing? 1) North 2) South 3) East 4) West Directions (14-16) : The following questions are based on the following diagram showing four persons stationed at the four corners of a rohmbus piece of plot. as shown. : A B 4. A starts crossing the field diagonally opposite. After walking half the distance, he turns right; walks some distance and turns left. Which direction is A facing now? 1) East 2) North 3) South 4) West 5. From the original position given in figure above. A and B move one arm length clockwise and then Crosses over to the corner dioganally opposite. X and Y, move one arm length anticlock wise and cross over the corner diagonally opposite. The original configuration AYBX has now changed to = 1) XBYA 2) EYAX 3) YAXB 4) AXBY 8. From the original position, B and Y move one and a half length of sides clock-wise and anti clockwise respectively. Which one of the following statements is true ? 1) BandY are both at the mid point between A and X 2) is at the mid point between A and X & B. One corner occupied by X 3) Bis at mid point between A and X and at the corner originally occupied by A 4)B and Y are both at the mid point between A and ‘2 O_O 17. A, B, C and D are playing carroms. A, Cand B, D are partners. ‘D' is to the right of C who is facing west. Then B is facing 1) North 2) South 3) East 4) West Directions Q.No. (18-23) : Six friends - Ram, Raju, Srinu, Vivek, Kiran and Vijay are sitting around a circular table all facing to the centre. Vijay is sitting to the left of Ram. Raju is sitting between Srinuand Kiran and is opposite to Ram. Kiran is sitting to the right of Vivek. 18. Whois sitting to the left of Raju ? 1) Kiran 2) Stinu 3) Vijay 4) Vivek 19. Whois sitting opposite to Vijay ? 1) Srinu 2) Vivek 3) Kiran 4) Raju 20. If each person interchanges their places with the person opposite, then who is Sitting tothe right of Srinu ? 1) Kiran 2) Raju 3) Vijay 4) None 21. If Ram interchanges. his place with the person sitting opposite to Vijay, then who sits to the right of Kiran ? 1) Raju 2) Srinu 3) Vivek 4) None 22. If Vivek interchanges his places with Vijay, then who is to the second left of Raju ? 1) Kiran 2) Vijay 3) Vivek 4) None 23. If Kiran and Srinu interchanges their places, then who is to the second right of Vijay ? 1) Vivek 2) Raju . 3) Ram 4) None Directions Q.No. (24-25) : There are five students P,Q, R, S and T are sitting on a Bench. P is at the extreme left, Q is second from the extreme right. T and Q are sitting together, T and R are sitting together, Q and S are sitting together. 24. Who is sitting between P and Q? 1) RandT 2)S andT 3) PandT 4) Can't say 25. Who is at the extreme right ? 1Q 2)R 3)s 4t 4 21 34 4)4 54 64 72 84 93 103 11)1 12)4 13)2 14)3 15)1 16)1 17)1 18)1 19)3 20)3 21)3 22)2 23)1 24)1 25)3 BLOOD RELATIONSHIIPS In this chapter, the success of a candidate depends upon the knowledge about various blood - lations, some of which are summarized below to help in solving the questions would be draw a tree agram as shown below. 1* Generation 2" Generation 3° Generation 4™ Generation 5™ Generation :|GRAND - PARENTS] - (Grand Father, Grand Mother) :|PARENTS and IN-LAWS] —> (Father, Mother, Uncle, Aunt, 1 Father-in-law, Mother-in-law) { SIBLINGS, SPOUSE &]-—> (Brother, Sister, Cousin, Husband, IN-LAWS. Wife, Brother-in-law, Sister-in-law) CHILDREN & IN-LAWS ]—> (Son, Daughter, Niece, Nephew, 4 :|GRAND CHILDREN |- (Grand son, Grand daughter) Son-in-law, Daughter-in-law) Given below the relations that which how they belong to yourself. 1. Mother's or Father's son - Brother 2. Mother's or Father's daughter - Sister 3. Mother's or Father's Brother - Uncle 4. Mother's or Father's Sister - Aunt 5. Father's Brother - Paternal uncle 6. Father's Sister - Paternal Aunt 7. Mother's Brother - Maternal uncle 8. Mother's sister - Maternal Aunt 9. Father's or Mother's sister's or Brother's Son or Daughter is called cousin 10. Mother's father - Maternal Grand father 11. Mother's mother - Grandmother 12. Father's father - Paternal Grandfather 13, Father's mother - Grand mother Eee aaa 14. Brother's or Sister's son - Nephew } 15. Brother's or Sister's daughter - Niece | 16. Brother's wife - Sister-in-law 17. Sister's Husband - Brother-in-law 18. Wife (or) Husband's Brother - Brother-in-law 19. Wife (or) Husband's Sister - Sister-in-law 20. Son's wife - Daughter-in-law 21. Daughter's Husband - Son-in-law 22. Grand Son's or Grand daughter's son - Great Grand Son 23. Grand Son's or Grand daughter's Daughter - Great Grand Daughter 24. Wife (or) Husbane's father - Father-in-law 25. Wife (or) Husband's mother - Mother-ir-taw 26. The only son of the grandfather - Father 27. The only daughter of the maternal grand father - Mother 28. Maternal uncle Father's only daughter - Mother 29. Son's son - Grandson 30. Spouse - Life Patner (Husband or wife) Example : Pointing to a man in a photograph Rahul said, "He is the brother of my uncle's daughter “Hews| is the man related to Rahul ? 1) Srather - in - law 2) Brother 3) Cousin 4) Uncle Answer : (3) , Srother | :XERCISE -7 Pointing to a photograph, a person tells his friend, "She is the grand daughter of the elder brother of my father". How is the girl in the photograph rolated to this man? 1) Niece 2) Sister 3) Aunt 4) Sister-in-law A women introduces a man as, “The son of the brother of her mother". Hows the man related to the woman? 1) Nephew 2) Son 3) Cousin 4) Uncle 3, While walking with his friend, Krishna meets another man “Whose mother is the wife of Krishna's father's only son*. How is the man related to Krishna? 4) Son 2)-Nephien, 3) Cousin 4) Father . Aman said to a lady, “Your mother's husband's sister is my aunt". How is the lady related to the man? 1) Daughter 2) Grand Daughter 3) Mether 4) Sister A girl introduced a boy as the son of the only daughter of the father of her maternal uncle. The boy is girl's 1) Brother ®) Sen 3) Uncle 4) Son-in-law i, If X is the brother of the son of Y's son, how's X related to Y? 4) Sen 2) Brother 3) Cousin 4) Grand Son Introducing a man to her husband, a woman said, "His brother's father is the only son of my grand father’. How is the woman related to this man? 1) Mether 2) Aunt 3) Sister 4) Grand Mother |. Pointing out to a lady a girl said, "She is the daughter-in-law of the grand mother of my father's only son", How is the lady related to the girl? 1) Sister-in-law 2) Aunt 3) Mother-in-law 4) Mother , Radha told a man, “The gitl | met yesterday at the beach was the youngest daughter of the brother-in-law of my- friend's mother". How is the girl related to Radha’s friend? 4) Cousin 2) Daughter 3) Niece 4) Friend ®. If'B' says that his mother is the only daughter of ‘A's mother. How is A related to B? 1) Son 2) Father 3) Uncle 4) Grand Father 1. Raghu told Praveen, “Yesterday | defeated the only brother of the daughter of my grandmother’. Whom did Raghu defeat? : Son Father 8) Brother 4) Father-in-law eR 12. When Ram saw Kiran, he recalled "He is the son of the father of the mother of my daughter". Who is Kiran? 4) Brother-in-law 2) Brother 3) Cousin 4) Uncle 13. Introducing a mana woman said, “He is the only son of my mother's mother’. How is the woman related tothe man? 1) Mother 2) Aunt 3) Sister 4) Niece 44. Pointing toa man ina photograph, Isha said, "His mother's only daughter is my mother". How Is Isha related to that man? 1) Niece 2) Sister 3)Wite 4) Nephew 15. Showing a lady in the park, Vijay said, "She is the daughter of my grand father's only son’. How is Vijay related to thal lady? 4) Brother 2) Cousin 3) Father 4) Uncle 46. Introducing man, a woman said; "His wife is the only daughter of my father’. How was that man related to the woman? 1) Brother 2) Father-in-law 3) Maternal Uncle 4) Husband 17. Ravi said to Nag, “That boy playing with the football is the younger of the two brothers of the daughter of my father's wife’. How is the boy playing fooiball related to Ravi? 1) Son : 2) Brother 8) Cousin 4) Nephew 18. Pointing to the lady on the platform, Mythri said, “She is the sister of ther father of my mother's son". How is the lady related to Mytiri? 1) Mother 2) Sister 3) Aunt 4) Niece 19. Balu said, "The girl is the wife of the grandson of my mother". How is Balu related to the girl? 41) Father 2)Grand Father — 3) Husband 4) Father-in-Law 20. Pointing to a man in the photograph, a woman said, "His brother's father is the only son of my grand father". How is the woman related to the man in the photograph? 1) Mother 2) Aunt 3) Sister 4) Daughter 21. Showing the man who is receiving the prize, Leela said, 'He is the brother of my uncle's daughter. How is the man related to Leela? = 1) Son 2) Brother-in-Law 3) Nephew 4) Cousin 22. Pointing to a person a man said to a woman. 'His mother is the only daughter of your father’. How was the woman related lo the person? 1) Aunt - 2) Mother 3) Wite 4) Daughter 3. Introducing a girl, Varun said, ‘Her mother is the only daughter of my mother-in-law’. How is. Varun related to the girl? 2 4) Uncle 2) Husband 3) Brother 4) Father 4. Pointing to a lady a man said, "The son of her only brother is the brother of my wife". How is the lady related te the man? 1) Mother's Sister 2) Grand Mother 3) Mother-in-law 4) Sister of father-in-law 5. Pointing to an old man, Jagdeesh said; 'His son is my son's unclo'. How is the old man related to Jagdeesh? 1) Brother 2) Uncle 3) Father 4) Grand Father 6. Cis brother of A; Bis daughter of A; Eis sister of C, and Dis the brother of B. Whois the uncle of D? 1A 2)B 3)c a ae 7. Qis brother of R; P is sister of Q;T is brother of S; S is son of R.Who are.the nephews of Q? 1) R&P 2)P&T 3)Q&T 4 S&T 8, Lakshmi and Meena are Rohan's wives. Shalini is Meena's step daughter. How is Lakshmi related to Shalini? 1) Sister 2) Mother-in-law 3) Mother 4) Step Mother '9. Gis A's father's nephew. Dis A's cousin but not the cousin or brother of C. How is D related to C? 1) Father 2) Sister : 3) Mother 4) Aunt 10. A party consists of grand mother, father, mother, and four sons with their wives and one son and two daughters to each of the sons. How many females are there in all? 1) 14 2)16 3) 18 4) 24 MRECTIONS : (Q.No. 31 to 34). Read the following information and answer the questions given below: ‘ A's the'son of B.C, B’s'sister has a son D and a daughter E. F is the meternal uncle of D. 41. Howis A related to D? : 1) Cousin 2) Nephew 3) Uncie 4) Brother 32. Howis E related to F? 4) Sister 2) Daughter 3) Niece 4) Wite 33. How many Nephew(s) does F have? —_1) Nil _2)0re 37. 35. 36. 39. 41. 42. . How is B related to E? 1) Uncle 2) Brother 3) Maternal Uncle 4) None P is son of Q while Q and Fi are the sisters. T is mother of R. If S is the son of T, which of the following statements Is correct? 1)T is the brother of Q 2) Sis the cousin of P 3) Q and S are sisters 4) Sis the Maternal Uncle of P A woman walking with a boy meets another woman and on being asked about her relationship with the boy. She said, "my maternal uncle and his maternal uncle's maternal uncle are brothers". How is the boy related to the woman? 4) Cousin 2) Son 3) Grandson 4) Husband Niraja who is Tarun's daughter, says to Deepa, "Your mother Rani is the younger sister of my fathar, who is the third child of Mr. Rao". How is Mr. Rao related to Deepa? 1) Uncle 2) Father 3) Grand Father 4) Father-in-law . Pisthe brother of D, Xis the sister of P. Ais the brother of F. Fis the daughter of D. Mis the father Of X. Who is the uncle of A? 1x ayP 3)F 4)M P is the brother of Q and R. S Is A's mother. T is P's father. Which of the following statements cannot be definitely true? 1) Tis Q's Father 2) S is P's Mother 3) Pis S's Son 4) Qis T's Son . Kis the brother of N and X.Y is the mother of N and Zis the father of K, Which of the following Statements is not definitely true? 1) Kis the son of Z 2) Y is the wife of Z 3) Kis the son of Y 4) Kis father of X Eis the son of A. D is the son of B. E is married to C. Cis B's daughter. How is D related to E? 1) Brother 2) Uncle 3) Father-in-law 4) Brother-in-law ‘and B both are children of C. If C is the mother of A. A is'the son of C but Bis not the daughter of C. How is A related B 2 1) Ais the brother of B 2) Ais the sister of B 9) Ais the cousin of B 4) Ais the nephew of B 47. 48. 49. 50. Directions:(0.No.43 to 48) Read the following information caretully and answer the questions given below: All the six members of a family'A,B,C,D,E and F are travalling together. B is the son of C but Cis not the mother of B. A and C are married couple. E is the brother of C. Dis the daughter of A. Fis the brother of E. . How many male members are there in the family? 1 23 3)2 44 Who is.the mother of B? 0) 2)F 3)E aA . How many children does.A have? 1) One 2) Two 3) Three 4) Four . Who is the wife of E? yA 2)F 3)B 4) Can't be determined Which of the following is a pair of Females? 4) AE 2) BD 3) DF 4) AD How is € related to D? 1) Father 2) Brother 8) Uncle 4) Can't be determined Madhavi told to Gouri, ‘The boy going on the motor bike is the youngest oi the three sons of the only sister of the eldest son of wife of my maternal grand father". The boy is Madhavi : 1) Uncle 2) Father 3) Cousin 4) Brother Lakshmi is the niece of the only sister of the ‘only son of the wife of Srinu’s grand father. How is Lakshmi related to Srinu? 1) Aunt 2) Mother 3) Sister 4) Grand Mother 411i 88 8)4 445) 6)4 73 84 Ot 10)3 12 12)1 13)4 141 15)1 16)4 17)2 18)3 194 20)3 21)4 22)2 23)4 24)4 25)3 26)3 27/4 28)3 -29)2 30)1 31)1 92)3 33)3.34)4 95)4 38)2 37/3 98)2 39/4 40/4 41)4 42)1 43)4 44)4 45)2 46)4 47)4 48)3 49)3 50)3 Analogy means similarity (or) correspondence. In each question we find two pairs. The first | pairin which there is some relationship oetween the components is called the ‘Head pair. One has to choose the required component of the ‘Question piar' from the options given so thst the similar relationship is maintained there too. We have in this topic questions related to both number and alphabet analogy. First, let us see some examples EXAMPLES : n:2n: Ex: 1)8:18::12: ge 1)24 2)30 Answer :(1) n:3n: Ex:2) 4:12:: 112 Answer : (4) n:Kn: Ex:3)8:40:: : 1) 10 2)23 Answer : (4) nin? : Ex:4) §:25::8:. - 1) 12 2) 64 Answer : (2) nim: Ex:5) 3:27::4:.. 112 Answer : (2) n:vn : Ex:6) 36:6::100:..... 1) 14 242 Answer : (3) 3)2 3)15 3)34 3) 36 3)35 3) 10 4)8 4) 24 4) 25 4) 72 4) 63 4)16 n:Vn: Ex: 7) 216; 6::512:.. 18 Answer : (1) minted Ex: 8)3:10: 1) 15 Answer : (4) nint-4 Ex: 9)5:24:: 1) 30 Answer : (3) ninten 1) 420 Answer : (2) nint—n Ex: 11) 9:72::6:. 1) 50 Answer : (1) nint+4 Ex: 12) §:126::8:.. 1) 500 Answer : (2) nin=4 Ex: 13) 4:63::10:.. 4) 200 Answer ; (4) ninen 4) 220 Answer : (1) nin—n Ex: 15)5:1207:6: 4) 200 Answer : (3) Ex: 10) 10:110::12:.. Ex: 14) 5:130::6:....... 2)9 240 2) 156 2)40 2)513 2) 300 2) 150 2) 150 3)10 3) 60 3) 63 3) 100 3)50 3) 400 3) 200 3)210 4a 4) 17 4) 70 4) 144 4) 60 4) 100 4)999 = 4)100 4) 100 nitns dy Ex: 16) 8:812:10°...... x 1) 100 2) 121 3) 125 4) 200 Answer : (2) n:(n-4)? é Ex NIG oye a ain 1)40 2)30 3)20 4) 36 Answer ; (4) ni(ne4)> Ex: 18) G2 2164s, = 1/125 2) 100 3) 50 4) 20 Answer : (1) n(n-1)5 Ex: 19) 6:64::4:. Bee 1)27 2)30 3) 40 ‘ 4) SC Answer : (1) ns (2nj* Ex: 20) 4:64::6: an 1144 2) 50 3) 100 4)70 Answer : (1) Ben 21) BeGibeh..... By 1M 2)Q 3)R 4T 5 letters are added to 'B' then we get 'G'. So, 5 letters should be added to 'L’ then we get 'Q’. Answer : (2) Ex: 22) Gu: KN::RU: < 1)VZ 2) WX 3) VY 4) WT Answer : (3) +1 1. ee 3:12::52:? 1) 202 16:49 :: 100° 7 , 722110: 169: 81::100:7 1) 64 16: 25::49:2 1) 63 10:26 ::50:7 6 87:52::69:7 181 .. Tae 7 1)70 ~ 20:90: 7:72 1)59 39: 68:7: 150 1) 101 . 56:90:79: 182 1) 132 1) 132 128i 7 64 1) 12 2) 104 2)144 2)82 2) 625 2)36 2) 36 2) 82 2)16 2) 86 2)81 2) 68 2) 105 2) 123 2) 182 2)16 EXERCISE - | 3) 204 3)81 3) 67 3) 125 3)72 3) 64 9) 122 3)25 3)88 3)69 3) 144 3) 126 3) 90: 3) 25 4) 208 4) 169 4) 101 4)25 4101 4) 36 4) 49 461 4) 121 4yi2i 4) 150 Her 3) 195 4) 145 35 41 9) 125 4) 150 2 2)4 36 4)8 47. 46:576::56:.. : 1) 525 3) 900 4) 125 48. 8:66::12: 1) 142 3) 146 4) 148 49. 196:14::961:. 121 3) 41) » 4) 51 50. 18:162::16: ; 1) 118 3) 156 4) 52 4 24 D2 ge yz ays Ho 83 93 ia WS 12)2 13) 141" 154 1€)3 A172 18)2 19}3 20)1 21)3 22)4 23)2 24)3 25)3 269 274 28)2 29)1 303 31)3 $2)2 33)4 3aj3 35)4 96)3 a7)d 38)1 39)4 anj2 41)3° 42)4 43)4 44)4 45)3 46)2 47)3 48)3 492 s0)2 tht 3) 65 4)70 a2 43 3) 72 4) 49 3) 83 4) 98 381 4) 83 3841 4) 144 3) & 4) 2100 3) 364.5, 4) 433.5 33 : 44 1)50 2) 100 370 4) 144 4) 3) 2764 492744 377 4) 60 sp aS ceeremrmnetenee UE BCD: FGH :: LMN:........... ACE : BDF =: HJL:? BYSC : YEHX |: MRCP: .. EGIK: FILO::; FHUL:.... ABEF : BCFG :: GHKL:....... PATNA : ANTAP :: KATNI! .....200.. BEAT :GIDV :: SOUP :... EHGTIRL PRS: >)... ). BACE: ONPR =: JIKM:.. » 2XYW: USTR=: PNOM: ........... » PRT: OGS:: KMO: ? LK IHG: FED: 2 . ACF :TVY ::BDG:? ALPHABET ANALOGY EXERCISE - 2 3)LM 3) STV 3) IKN 3) NIXK 3) JGMP- 3) HIUK 3) ITNAK 3) YSXR 3) TUX 3) XYWZ 3) JHKI 3) JNM. 3) DCB 4) HL 4) Xwv 4) KMN 4) O1YM 4) JGPM 4) HIML 4) INKTA 4) XSYR 4)wvl 4) WVXZ 4) HK 4) JLM 4) CBA 4) XYZ anise Orley i Saeed 15. EGI: QSU ':FHJ:? 1) PRT 2) RTV 3) PQS 4) RTU 16. PNL: OMK::?:XVT 1) XYW 2) BAZ 3) YWU. 4) VUW 17. FGH:KLM::?: STU 1)MNO 2) NOP 3) NOQ é 4) RST 18, WVU: ROP: ?: MLK 1) RAP. 2) POR 3):PRT 4) QST 19. LMN: QRS ::?: XYZ 1) PST 2) STV 3) STT 4) STU 20. DEF : JKL :?:TUV 1)PTO 2) NOT 3) NOP 4) POT 21, ACE: FHJ::0QS: “Sip 1) PRT 3) TVX 4) RTY 22. ABD: CDF::MNP: < 1) OPR 2) NPQ 3) VXY 4) LNO 23. UTS: EDC ::WVU:.. ieee 1) WXY 2) XW 3) SRP 4) RPO 24. Mgk:nHL:: ime 1) Lib 2) kgB 3) LRB 4) LB 25, TREE: RTEE::WANT: 1) NTWA 2) AWTN 3) TNAW 4) None 26. BORN:YLIM::FIRST:..... e 1) URIHG 2) GKPWY 3) RUIHG 4) None 27. WORLD :YRVQU:: PEACE: : : 1) RHEHK 2) SHEHK 3) REHHK 4) None 28, BDL:GIQ::APN:.......... ite 1) GVT 2) FVT 3) FIV 4) GTV 29. CIRCLE: RICELC::POCKET:... 1) CKOPET 2) CPOKET 3) COPTEK 4) None eb 53 ). EVTG: HSQJ:: CXVE? 3.2 ‘ ‘ 8) FUHU ee 1) FUST 2) FUSH . PINK: KNIP.:: BROWN :.. alate: 1) NBROW 2) MOWBR 3) NOWRB . COOL: ABNPNPKM: :HEAT ..... a 1) GIDFBSUI 2) MGEBCUVW 3) GIDFZBSU . ACEG : DEHN: QSUW? access 4) TWYZ 2) TVXZ 3) KMINP. . PKDL: QUFJ::OMLX:. Ze 1) MONV 2) PLNV 3) NLMU . CAT: DDY :: MAT 4) NDY 2) NDX 3) DDY . SEUQCAJ: LCESWGU: : ECNARFP :.. 1) DBMZQEO 2)FDOBSGQ 3) GEPBSGR . SAMOHT : SINNZT: : RELHEM : o 1) QFKIDN 2) SFMIFN 3) QDKGDL . SQEOIA : ZHNDPR ::REZTIW: 3 1) VHUADQ 2) RUSYFS 3) VHSYDQ . NASUBL: MBRVAM : :«..sases:sesetts et DMRFUS 1) CNQGTT 2) ELSEVR 3) ENSGVT . NOITGC; OPHSHD ::... ..: MBMQPV 1) NCLPQW 2) LALRQU 3) LANROU 1)3 2)4 3)2 44 5)2 6)2 13 8)4 92 11)2 12)1 13)4 14)3..15)2 16)3° 17)2..18)1 19)4 21)3 22)4-.23)2- 24)4 25)2 26)1 27/1 282 2973 31)4 32)3 33)2-34)2 -35)1 98)4 «37/4 «38)3 39)2 4) FSUH 4) NWORB 4) GHBCDESR 4) MNPR 4) PLMV 4) OEX 4) RHTCPEG 4) LF@MDS 4) RHUYEQ 4) CLQETR 4)NANPQU 30)1 20)3 30)2 40)3 rs CLASSIFICATION (ODD MAN OUT) ‘Classification’ means to assort the items of a given group on the basis of certain comma, quality they posses and then choose the stranger one out. Examples On Number Classification : Ex: 4) -2,3,5,7,9, 11,13 1)3 2)5 3)9 4)13 Answer ; (1) All the numbers except 9 are prime numbers. Ex: 2) 1,4,9, 16, 25, 36, 48, 64, 81 1/16 2)36 3)48 4) 81 Answer : (3) Except 48 others are squares of natural numbers Ex:3) 4, 16, 36, 64, 100, 121, 144, 196 1) 122 2)64 3) 100 4) 144 Answer ; (1) Except 121, others are squares of even numbers Ex: 4) 1,8, 27, 64, 124,216, 349, 512, 729 1) 64 2) 124 3)216 4) 729 Answer : (2) Except 124, others are cubes of natural numbers Ex:5) 4,9, 16, 25, 49, 121, 169 1) 16 2) 25 3) 121 4) 169 Answer : (1) others are squares of prime numbers. Remark : The following lagical sequence avoids all sorts of contusion in identifying the suitable option. Prime number logic > cubic logic > square logic > (multiplication / division) logle » (addl- tlon / difference) logic. etter ification Letter classification problems are almost similar ta letter analogy problems. Tho following table will help the candidates to solve the letter classification problems. Creative Learning Institute for Comprehensive Knowledge : cCLiCcK® EXAMPLES: Ex: 1) Two letter Classification : 1) AZ 2) BY 3) Cx 4) ET Answer : (4) Logic : In all other cases, if the letters are written in reverse order, their positions are same Ex : 2) Three letter Classification 1) ABD 2) EFH 3) WK 4)LMO. Answer ; (3) Logic : In all other terms, the gap between first, second and third letters is the same. Ex : 3) Four letter Classification 1) PRTV 2) JLNP- 3) BDFH 4) ABCE Answer : (4) Logie : In all other cases the gap between any two successive letters is the same. (One latter is skipped) Ex: 4) Mixed Classification I A27Z 2)022G 3)U23B 4) M28 J ‘Answer : (3) Logic : Ifnumbers are assigned to the letters, the sum of the numbers of the letters gives the number. Similarly, a number of other types of letter classification problems can be identified, ‘ord Classification Here, three of the given four items are based on a common Property (for example : name, Places, professions etc.). We have to pick the thing which does not passess the same property. XAMPLES ; Ex: 1) 1) Apple 2) Banana 3) Rose 4) Orange Answer : (3) Logic : Except ‘Rose’, all other are fruits. Ex :2) 1) Metal 2) Gold 3) Silver 4) Platinum Answer : (1) Logic : Except ‘Metal’, all the three are different forms of metal. EXERCISE Directions : Three of the given four items in each of the following question have the same relatior ship. Pick the odd thing out. 1) 24 2).60 9) 124 4)210 2. 1144 2) 169 3) 256 4) 324 Bo aes. 2) 343 3) 729 4) 1331 | 4. 1)36 2)72 3) 96 4)85 5. 1)16-29 2) 95-48 3) 49-61 4) 78-91 6. 1) 13,2629 Q)15.30.32 3) 18,3639. 4)23,46.49 7. 1) 164 2) 134 3) 184 4) 192 8. 4/35, 271 3) 44 4) 63 97 1)25 2) 48 3)64 4)16 10. 1)56 2) 65 3) 85 4)75 11. 1)242 2) 363 3) 121 4)585 12 1717 2)39 3)37 4)59 18. 1)56 2)80 3)96 4)44 4. 1)36 2)96 3) 28 4)72 15. 1)99 2)198 - 3)396 4) 592 16. 1)134 2) 628 3) 289 4) 538 17, 1) 2844 2) 6234 © 3)8136 4) 1643 18. 1)353 2)929 3)212 4) 837 19. 4) 101 2) 808 3) 11011 4) 909 20. 1)222 header 3) 165 4) 242 21. 1) 144 2) 264 3) 384 4) 500 22. 1)1401 2) 1001 ~ 9)1014 ain 23. 1)567 2)789 | 3) 456 4) 148 24. 1)96 2)78 3) 64 4) 48 25, 1)43 2) 53 3)63 4)73 26, 1) 45 2) 406 3) 205 4) 648 27. 1) 1342 2) 6028 3) 4562 4) 3245 28. 1) 121 2) 144, 3) 169 4) 196 29. 1) 842 2)933 ° “3)633 4) 824 Creative Learning Institute for Comprehensive Knowledge RROD ACR A Ra Sra 70. 1414 2) 1)732 3)2 4) 3)5 31. 1)225 2) 2025 8) 5025 4) 9025 92. 1)192 2) 156 3) 182 4) 242 33. 1)459 2) 234 3) 567 4) 242 34. 1) 64 2)81 8) 100 4) 144 35. 1)234 2)345 3) 456 4) 687 36. 1)385 2) 132 3) 5116 4) 3119 37. 1)104 2) 111 3) 121 Oyiae hs 38. 1)121 2) 444 3) 991 4) 1681 39, 1)32 2) 24 3)6 4)12 40. 1) 15 2)35 33 4)12 41. 1) AEFU 2) KOPT 9) uyzD 4) EHIL 42. 1) ACE 2) MOQ 3) JLN 4) STV 43. 1) BGL 2) HMR 3) QvY 4) KPU 44. 1) DF 2) LN 3) RT 4) CA 45. 1) AN 2) DQ 3) GT 4)NZ 46. 1)MNPTB 2) ABDHP 3). GHINT 4) YZBFN 47. 1)DIN 2) FED 3) OTY 4) JOT 48. 1) URX 2) KHN 3) QMT 4) DAG 49. 1) TRV 2) QOR 3) VTX 4) DBF 50. 1) BEDC _ 2) HK 3) TWvu 4) PSRQ 51. 1)FR S 2)DL 3) NV 4)1Q 62. 4ySv yur 3) CK 4) OW 53. 1) EICD 2) MQKL 3) sSwaR 4) PURS 54. 1) AF 2) DI 3) EA 4) KP 55. 1) ADG _ 2)BEH 3) Zwt 4) CFI 56. 1) DCBA 2) HGFE 3) MRUX 4) PONM 57. 1)AIQ 2) IMIN 3) BUR 4) CKS 58. 1) ADH 2) CFU 3) JNR 4) BEI 59, 1) AFJC 2) FKOH 3) QVZS: . 4) IMOK 60. 1) Bei 2) Ciy 3) Dul 4) DTm 61. 1) RRQ 2) OMN 3) TPS 4) FDE Creative Learning Institute for Comorchensive Knowledae : CLICK 62. 63. 64, 65 66 67) 7 70 A 72. 73. 74, 75. 76. 7. 78. 79. 80. at. 82. 83. 84, 85. 86. a7. 88. 89, 90. gg: 1) ABC 1) NRV 1) ZTAX 1) Moa 1) BFH 1) GHRS 4) DI6P. 4) BL 1) EC 1) OTP 1) BCE 4) RUX 1) CEI 1) JOT 1) FAA 1) MP 4}U 1) PQs 1) BOE 1) Cricket -1) Chair 4) Black 4) Man 1) Man 1) Earth 1) Circle 1) Rice 1) Sugarcane 1) May 1) Sunday 1) Oil 1) Roof 2) JKL 2) DLT 2) TAZX 2) jLN 2) NRT 2) PQAB 2) Cal 2) KU 2)TR 2) AFB 2) HKA 2) GUM 2) PRU 2) OUT 2) OFF 2) BE 2)V. 2) AB 2) XpD 2) Chess 2) Sofaset 2) Black Smith 2) Animal 2) Wife 2) Moon 2) Rectangle 2) Wheat 2) Tobacco 2) June 2) Week 2) Coal 2) Wall 3) ONM 3) AKU. 3) XZTA 3) eGl 3) GJL 3) BDIJ 3) D25Y 3) PY 3) LN 3) SXT 3) LMO 3) NPS 3) JLP 3) FED 3) ATT 3) UR 3)T 3) SnM 3) HQu 3) Volley Ball 3) Carpet 3) Red 3) Plant 3) Husband 3) Mercury 3) Triangle 3) Peanut 3) Tea 3) Summer 3) Monday 3) Peat 3) Window Creative Learning Institute for Comprehensive Knowledg ni: 4) POR 4) OMS 4) AXZT 4)iTQ 4 4) MQS 4) DELM 4) BaD 4) OY 4) US 4) UVB 4) PAS 4) ZCF 4) QSW 4) DIN 4) FF 4) FI 4a)x 4) mnZ 4) Mkv 4) Foot Ball 4) Bench 4) Green 4) Soil 4) Sister 4) Venus 4) Square 4) Gram 4) Rice 4) July 4) Friday 4) Coke 4) House 3) Nice 94. 95. 96. 97. 98. 98; 100. 1) Good 1) Replup 1) Fear 1) Floppy 1) Tarapur 1) Bigger 1) Mansion ya 22 114 12)2 21)4 222 31)3 324 44)4 4a)4 St)1 52)1 61/3 62)3 7iy4 72)2 81)2 82)3 91)2 92)4 33 13)1 23) 4 33)4 43) 53)4 63)4 73)3 83)2 93) 4 2) Bad 2) Koyned 2) Love 2) Windows 2) Kota 2) Faster 2) Apartment aA 44)1 24)2 34)2 44) 4 54)3 64) 2 7A) B4)4 94)2 5)3 15)4 25)3 95)4 45)4 55)3 65) 4 75)2 85)1 95)2 6)2 16)3 26)3 36)4 46)9 56)3 66)3 76)3, 86)2 96)2 3) Loitev 3) Prido 3) Eolair 3) Kalpakkam 3) Greater 3) Garrage 12 ine 27)3 37) 11 422 57)2 67)3 77)3 87)1 97)4 ooo a2 18)4 28)4 38)3 48)3 58)3 68)3 78)1 88)4 98) 4 92 19)2 29) 39)2 49)2 59) 4 69)3 79)4 89)4 99)2 4) Better 4) Georna 4) Sorrow 4) Fortran 4) Paradeep 4) Largor 4) Villa 10)1 20)1 30) 4 40)2 50)2 60)4 70)3 80)3 90)3 190)3 ing sald hachin Foe: Comprehensive Knowledge : CLICK N 60 SMES FU a Tea vale. SYMBOL. [D NOTATIONS In this type of questions we deal with simple Mathematical operations. i.e. like Addition, Subtraction, Multiplication, Division and Statements such as less than '<', greatorthan >', equal to'~", notequalto' < ‘etc. Replace the symbols by mathematical operaitons and apply 'BODMAS' rule to set the suitable answer. B = stands for Bracket [{( )}] O = of (nothing but x) D = stands for division (+) M = stands for Multiplication (x) A = stands for Addition (+) S = stands for Subtraction (-) \f- means X, X-means +, + means + and + means -, then what will be the value o 40x124+3-6+60=? 1) 44 2)15 3)16° 44 If + means +, + means X and X means +, then 48+6x4 + 28=7 1)110 e 2) 115 3) 120 4) 140 If+ means X, X means +, + means - and- means ~, then $x 6-12x1-2+9 ys 2) 7/24 » 3) 25/8 4) 3/4 If + means +, + means -, - means X and X means +, what will be the value of the following expression 8 +4+3x5-9=7 1)44 2)5 2/3 2618 4)46 If + means +, - means +, X means - and - means X, then 8+ 4-6 +3x4=?7 18 j 2)46 3)4 4330 f+ means +, X means -, + means X and - means +, then 94+ 3+4-8x2=? 1) -6 1/4 2)6 3/4 3)-19/4 418 If + means X, - means +, X means - and + means +, then 9 + 8+ 8-4x9=? 1) 26 2)61 3) 65 ayy If + means X, - means +, X means +, find the value of 54+4-18x3=? 1} -34 2)6 3)26 4) 14 if + means +, - means X, + means - and X means +, what will bo the value of 8+ 6+ 4-7x3=9 1) -23/2 2) 14 3)-71/3 4)12 61 BRAM bedee th belt aek eh L=3 0. If A stands for +, B stands for -, C stands for X, wnat is the value of (10 C 4) A(4.C 4) B6=7 1)46 2)50 3) 56 4) 60 1. If+ means +,-means+, X means - and + means X, then (36x4)-8 x4 /4+8x3416+1=? 1.16 2yi2 38 40 2. If X stands for ‘add’, Y stands for ‘subtract’, Z stands for 'devide', P stands for ‘multiply’ then find the value of 10x 10y5z5 =? 1 2)20 3) 30 419 8. If+ stands for multiplication, - stands for division, x stands for addition and + stands for subtraction, then value of 5 +6x8-2+3=? 1) 83 2)6 3)31 4) 16 4. If + stands for ‘division’, - stands for ‘equal to’, x stands for ‘addition’, + stands for ‘greater than’, = stands for ‘less than’, > stands for ‘multiplication’ and < stands for ‘subtraction’, then which of the following alternatives is correct? 1) 542x1=34451 2) B>2u1-Godel 3) SD t=4x1 5. IPP denotes =, Q denotes x, R denotes + and S denotes - then 18 Q 12P4R5S6=? 1) 10 2)53 3)56 4) 57 5. If L denotes x, M denotes +, P denotes + and Q denotes - then 16 P24M6Q6M2L3 =? 4) 13/6 2)-16 3) 14172 4)10 7. IFA denotes -, C denotes x, D denotes + and E denotes +, thon 14 3A 12E4D2=? 1)17 2)32 3) 28 4)6 3. If J denotes +, K denotes x, L denotes - and M denotes + then 9M 14J7L8K2=?7 4)6 2) -89/7 3)5 4) -23/9 9. IP denotes +, Q denotes -, R denotes x and § denotes + then8 R&P.8S8Q8-? 4) 10 2) 100 3) 56 4) 87 >. If+ means +, x means -, + means x and- means + then 8+ 15 -3x442=?7 418 z 2)5 3) 23/24 4) 23/6 4 23 °3)1 41 94 4 NS 3 Q3 OE 11)4 12)4 13)3 14)2 15)2 16)4 17/2 18)3 19)4 20)2 es eal ALPHABET TEST DIRECTIONS Q.NO. (1-15): ABRANGE THE GIVEN WORDS IN ALPHABETICAL ORDER AND CHOOSE WHICH COMES SECOND 1. 1) NATIVE 2) NARRATOR 3) NAUGHTY 4) NATURE 2. 1) TEMPERATURE 2) TEMPERMENT — 3) TEMPTATION 4) TEMPO 3, 1) PICTURE 2) PIC 3) PIN 4) POT 4. 1) ASSISTANT 2) ASBESTAS 3) ASS 4) ASSESSMENT 5. 1) EVERY 2) ENTER 3) EARNING 4) ELBOW 6. 1) HELLO 2) HOST 3) HEN 4) HAND. 7, 1) SPOUSE 2) SPORTSMAN 3) SONG 4) SPORADIC 8. 1) LEGAL 2) LEGACY 3) LENGTH 4) LENIN 9. 1) BOOST 2) BOOM 3) BOMMER 4) BASKET 10. 1) GALLON 2) GOAT 3) GRAB 4) GHOST 11. 1) FILTER 2) FROG 3) FEAR 4) FILLING 12. 1) INDIA 2) INK 3) INDIGO 4) INDICA 13, 1) ZEBRA 2) ZERO 3) ZEAL 4) ZING. 14, 1) LATTEST 2) LATTER 3) LAST 4) LATE 15. 1) QUATER 2) QUALIFY 3) QUANTITY 4) QUALITY DIRCTIONS : If itis possible to form a meaningful word by using the letters of the given words, writ the first letter of that word, If it is possible to form more than one word, give your answer as ') and no such word is formed give your answer as 'Y": By making use of 1% ,2"¢, 3r¢ and 4! letters in" STAINLESS" IA 2s 3)X ayy By making use of 2", 4" and 6" letters in "FORTUNE" 1)N 2)T 3)X ayy . By making use of 4", 6, 8", and 10™ letters in "MAINTANANCE" a)A 2c 3) 4)y . By making use of 4%, 8" and 9" letters in * MISFORTUNE” SNS 2)F 3) 4)y . By making use of 24, 6, 8", 9", 10" and 13" lelfers in. * ANDHRA PRADESH" 1)A 2)R 3) ay “Which letter i is eighth es ine vett = ‘Sixteenth fetter aoe right end of the alphabet?’ 1)D QE ac A)F .. Which letter is sixteenth to the right of the letter which is fourth to the left of L? 1Y 2)x 3)w au .. Which letter is midway between Fifteenth letter from left and eighteenth letter from right end of the alphabet? 10 2yP aL aN . Which letter is seventh to the right of the thirteenth latter from the left end of the alphabet ? ys 2)T 3)R : AU .. Which letter will be the ninth to the right of the seventeenth letter from the rightend of the alphabet? is 2)R aT 4)Vv Which letter is Sixth to the right of eleventh letter from right end of the alphabet? 1)U 2)v 3)w ax . In the alphabet which letter follows the 12th letter from left and preceeds 13th letter from right end? 1 2)M 30 aT . Which fetter is midway between the 10th{etter from the right and seventh letter from the left in the alphabet? 1)B 2)D 3)L 4)M . Which letter is 6th to the right of 7th letter from right end of the alphabet? 1)8 2A 3)Z 4)D . Which letter is 7th tothe right of 12th letter from right end of the alphabet? 1) 2)Xx aw av . Which letter is eleventh to the right of 12th letter from left end of the alphabet? 4)Q 2)T 3)R 4)W 82. Which letter is 13th to the right of Sth letter from left end of the alphabet? ns 2)v 3)R 4)N 33. Which letter is 15th to the right of 6th letter from left end of the alphabet? ye 2uU 3)0 4)K 24. Which letter is 6th to the left of 14th letter from right end of the alphabet? 1F 2)J 3)G 4)H 35. Which letter is 7th to the right of 8th letter from right end of the alphabet? 1Y 2)Z 3) X 4)w 36. Which letter is 7th to the right of 15th latter from right end of the alphabet? yt 2S 3)R 4Q 37. Which lettor is 10th to the left of 12th letter from left end of the alphabet? 1A 2c 3)B 4x 38. Which letter is 9th to the right of 15th letter from left end of the alphabet? 10 2)P. 3)R 4)x 39. Which letter is 10th to the right of 15th letter from left end of the alphabet? 1)Vv 2Y ' 3)w 4)X 40. Which letter is 10th to the right of 17th letter from right end of the alphabel? NT 2)R 3aQ 4p 41. Which letter is 11th to the left of 12th letter from right end of the alphabet? 1A 2B 3)D 4c 42, Which letter is Sth to the left of 20th letter from right end of the alphabet? 1c 28 3)A 4z 43. Which letter is 18th o the let ot ath fatter from right end of the alphabet? 1A 2a 3)P aR 44. Which letter is 12th to the right of 10th letter from left end of the alphabet? aL 2)Vv 3)T 4c 5. Which letter is 5th to the left of 15th letter from right end of the alphabet? 1H 2b 3)G 4)M 6. From the word 'LAPAROSCOPY’, haw many independent Meaningful words can be made without changing the order of the letter$ and using each letter only once’? 1) One 2) Two 8) Three 4) More than Four ._ If the first and second letters in the word 'EARTH QUAKE’ were interchanged, also the third and fourth letters, and the fifth and sixth letters so on, which of the following would be the seventh letter from the right? )T 2H 3)R 4A 8. How many pairs of letters are there in the word ‘MASTER’ which have as many letters between in them in the word as they have in the alphabet? 1) One 2) Two 3) Three 4) None 9. How many pairs of letters are there in the word 'CONVECTION’ which have as many letters in between them in the word as they have in the alphabet? 1) One 2) Two 3) Three 4) More than Three 9. How many pairs of letters arc there in the word 'MOTHER LAND' which have as many letters in between them in the word as they haye in the alphabet? 1) One 2) Two 3) Three 4) None 11 -2)2) 8)t 484 BT A 81) 3104 W1)4 12)4. 48)1 14)4 15)4 16)2 1793 18)4 1992 20)1 21)3 22)2 23)3 24)2 25)1 26)2 27)2. 28)3 29)8 a0)4 31)3° 32)3 33)2 34)3 35)2 36)2 37/3 38)4 39)2 40)1 41)3 42)2 43)1 44)2 45)3 46)2 47)3 48)2 49)1 50)2 te? Directions : In this type of questions, generally there are given a long series of numbers or alphabets. The candidate is required to find out how many times a number satisfying the conditions, specified in the question occurs. 41. How many 7's are there in the following number series, which are preceeded. by 1 bui nol followed bya? : 242178317298717832173423112 1) One 2) Two 3) Three 4) Four | 2. _ How many 2's are there in the following number series, which are preceeded by an odd number but not followed by an even number: 224352862327529122132912 1) One 2)Two 3) Three 4) Four 3. How many 1's are there in the following number series, which are preceeded by itself and followed by itself? 35121112111428191161211189 1) One 2)Two 3) Three 4) Four 4. How many even numbers are there in the following number series, which are preceeded by an even number and followed by an odd number? 86768932753422355328335 1) One 2)Two 3) Three 4) Four 5. How many 7's are thera in the following serias which are preceeded by 6 which Is not preceeded by 8? 87678675679761677688697687 1) One 2) Two 9) Three 4) Four 6. _ Inthe following series, how many 3's are followed by 1 bul not preceeded by 5? 5313153113555133131553153131351531 1) Two 2) Three 3) Four 4) More than four 7. How many 6’sare there in the following number series which are preceeded by 8 butnot immediately followed by 2? 408853126525465621 8526274851 1) Two 2) Three 3) Four 4) None Creative Learning Institute for Comprehensive Knowledge : CLICK 67 TS ear Lil Cc) How many A's are there in the following letter series which are followed by B but not preceeded by C? MBAMCABDABBMDACMABMPBDAGBCMAB 1) One 2) Two 3) Three 4) Four How many D's are there in the following letter series which are followed by L but not preceeded by M? ABCDDLPMDOLLADLCABBCDLPDM 1) One. 2) Two 3) Three 4) Four ), How many M's are there in the following letter series which are followed by N and also preceeded by N? NMNNNMNMNNNMNNMMNMNMNNMNMN 1) One 2) Two 3) Three 4) More than Three . Inthe following series, how many such odd numbers are there which are divisible by 3 or §, then followed by odd number and subsequently followed by even number too ? 12, 19, 21, 3, 25, 18, 35, 20, 22, 21, 45, 46, 47, 48, 9, 50, 52, 54, 55 1) ONE, 2) Two 3) NIL 4) NONE irections Q.No. (12 - 32): In each of these questions, the ranking of persons in a row from either ‘treme or both are given. Some of them may interchange their positions also. Identify the suitable dtion according to the instruction given in the question. 2. In a row of 20 boys if Rahul shifts 5 places to his left he becomes 13" from the left. What is his previous position from the right end? aan pane aye" 4) None . Ina row of people Priya ig 10th from the right end and 28th from the left end. How many people are there all in all in the row? 1)36 2) 37 3) 38 4) 39 |. Bala is 16th from either end of a row. How many students are thera in the row? 1)92 2) 33 3) 31 4)30 5. Ina row of people Sridhar is 22nd from front and 29th from back. How many people are there in the'row? ° 1)52 2)51 3) 53 4)50 . Rani got 12th rank in maths. Her place is 32nd from last, How many students are there in the olass? 68 BL Seu they 17. Rajesh is 42nd person to receive the sports award. His place from the last also is 42nd only. How many people received the award intoto 2 1) 82 2) 83 3) 81 4) 80 18. Swathi is 33rd in a row from the left end. Her place from the right end is 44th, How many persons are there in the row? 1)73 2)74 3)75 4) 76 19. If Rahul is 15th from the back in a row of 80 persons, what is his position from the front? ’ 1) 65 2) 67 3) 66 4) 68 20. Ina row of 25 people, if Sridevi is 13th from the left end, what is her position from the right and? 1)12 2)13 3) 14 4)15 21. If Arun is in 4th tank in a 50 member class, what is his position from the last? 1)47 2)48 3) 46 4)45 22. If Sindhu is Sth in the world ranking of 35 singers in the world, what is her position from the last? 1) 28 2)26 3) 27 4)30 23, Raviand Rohan are 7th from the left and 8th from the rightrespectively in a row. If they interchange their positions Ravi goes to 13th from the lett end. How many people are there in all ? What is the new position of Rohan from the right end? 1) 20, 15 2) 20,14 9) 21,14 4) 21,15 24. Chander and Kiran are respectively 10th from the left and 16th from the right in @ row. If they mutually interchange their positions, Chander comes to 21st from the left end.What is the strength Of the row? What is the new position of Kiran from the right end? 1) 36, 26 2) 35,27 3) 36, 27 4) 36, 28 26. Eswar is 12th from the leftend of a row, Farug Is 16th from the right and of that row: When they interchange, Eswar is 20th from the left end. How many people are there in the row? Whatis the new position of Farug from the right end ? 1) 36, 25 2) 36, 24 3) 36, 25 4) 36, 24 26. Anji and Bhanu are 9th from left and 10th from right respectively in a row. If they interchange thelt Positions Anji goes to 15th position trom the left end. How many people are therein all? What is the position of Bhanu from the right end? 1) 24,16 2) 25,16 . 3) 24,17 4) 24,15 sahehl beh ea Ld cd In @ row Ganesh is 17th from left, Harish is 20th from right. When they interchange Ganesh comes to 33rd position from the left. How many people are there in all ? What is the new position of Harish from the right end? 1) 50, 36 2) 53, 36 8) 52,36 4) 51, 36 Indra.and Varuna are 7th from the left and 13th from the right respectively in a row of Gacis. When these two got interchanged their positions, Indra goes to 15th position from left end. How many Gods are there in all? What is the new position of Varuna trom the right end? 1) 28, 21 2) 27,21 3) 27, 22 4) 27, 23 A & Bare 6th from the top and 13th from the bottom in a row respectively. When thoy interchange Als 20th from the top. How many people are there in all? What is the new position of B from the bottom? 1) 92, 28 2) 82,27 3) 32,26 4) 33, 26 In a row of girls Seetha and Radha are 14th from left and 20th from right respectively. When they interchange their positions Seetha is 20th from left. How many girls are there? 1)29 2)39 3) 38 4)37 In a row of books 10th one is a History book from the front, 12th book from the back is a Physics book. When these two books are interchanged History book is 20th from the front. What Is the total number of books in the row ? 1)29 2)30 aan: 4) 32 A & Bare 12th from the top and 15th from the bottom respectively. When they interchange A is. 30th from the top. What is the total number? What is the new position of B from the bottom? 1) 44, 32 2) 45, 33 3) 44,33 A) AA, 31 Among 5 friends Kiran has less income than Chanti, but more than Bhanu. Devan earns more than Chanti out not as much as Eswar. Who has the highest income? 1) Chanti 2) Bhanu 3) Devan 4) Eswar Ais taller than B, Bis taller than C, Eis taller than D, Dis taller than A. Identify the tallest and the shortest respectively ? 1)Eandc 2)DandE 3)BandA 4) None Ameer is older than Syam but younger than Chandu. Srinu is younger than Eswar but older than Kiran. It Kiran is older than Syam, who is the oldest? 1) Balu 2) Chandu 3) Anand 4) Data insuflicient Abhi is taller than Ramu, Raju ‘s talier than Abhi, Chandra is shorter than Ramu. Who is the tallest? 1) Abhi 2) Ramu * 3) Raju 4) Chandra REASONING 70 37. Raviis taller than Vijay, Rajuis taller than Syam but shorter than Bala. Vijay is shorter than Syam. Syamis taller than Ravi. Who is the tallest? 4) Vijay 2) Ravi : 3) Bala 4) Syam 38. Bis twice as old as A but twice younger than F.C is half the age of A but is twice older than D. Who is the youngest and who is the oldest? 1)D &A 2)BaC 3)A&B 4)D&F 39. Maruthi is taller than Srinu. Anil is taller than Maruthi. Ramesh is taller than Anil. Praveen is the talloost of all. Who is exactly in the middle of the order ? 1) Maruthi 2) Anil 3) Srinu 4) None 40. Sravya is wiser than Ramki. Kiran is wiser than Priya. Keerti is wiser than Tara. Ramk is wiser than Keerti, Priya is wiser than Sravya. Who is the wisest of all 2 1) Kiran 2) Sravya 3) Ramki 4) Keerti 12 23 9)9° 4)4 5)3 6)3 7/2 84 93 10)4 11)4 12)3 13)2 14)3 15)4 16)1 17)2 18)4 19)3 20)2 21)1 22)3 23)2 24)3 25)4 26)4 27)3 28)2 29)2 30)2 31)3 32)3 93)4 94)1 95)4 96)9 97)3 38)4 30)2 40)4 +? Ce I ll ae I OO i ia a br tad TIME SEQUENCE (Arrivals, Departures and Schedules) Vinayak says that the birth day of his brother Kalyan is before 20th January and after 17th January. While his mother remembers that Kalyan's birth day is after 18th January but before 21st January, on which date of January is the birthday of Kalyan? 4) 18th 2) 22nd 5 3) 19th 4) None If the seventh day of a month is three days earlier than friday, what day will it be on the 20th day of that month? 1) Sunday 2) Monday 3) Tuesday 4) Wednesday if the 3rd day of the month is Wednesday, which of the following will be the fifth day from 18th of that month? 1) Monday 2) Tuesday 3) Wednesday 4) Thursday Sanjeev remembers that his sister Rani's birthday is after 10th of March but bofore 16th of March. Whereas his father remember that his sister Rani's birthday was after 14th March but before 18th March (both are correct). Then on which date of March the birthday of Rani's falls ? 4) 12th 2) 13th 3) 46th 4) 15th if the day before yesterday was Monday, when will be the Thursday ? 1) Today 2) Two day after today 3) Tomorrow 4) Day after tomorrow ‘Adarsh went to market nine days ago. He goes to the market only on Friday. Whal day of the week Is Today? 1) Tuesday 2) Wednesday 3) Saturday 4) Sunday ‘Abus for Chennai leaves every 30 minutes from a busstand. An enquiry clerk told a passenger that the bus had already left 10 minutes ago and the next bus will leave at 9.35 a.m. At what time did the enquiry clerk give this information to the passenger? 1)9.10a.m. 2)8.55 a.m. 3) 9.15 a.m. 4) 9,05 a.m Ata Railway Station a man said to Kiran. "A train leaves for 'Vijayawada' for every 65 minutes. The last train has already left 25 minutes ago and the next train will loave at 8.15 p.m." At what time this information was given to Kiran? 1) 7.55pm. 2)7.50 p.m. 3)7.45 p.m. 4) 7.35 p.m. Enouiry clerk at Jublee Bus Stand said to Vishnu, "A bus for ‘Warangal’ leaves after every 55 minutes. The last bus has already left 20 minutes ago and the next bus will leave at 11 a.m." Al what time did the enquiry clerk gave this information to Vishnu? 1) 10.25 a.m. 2) 10.30 a.m. 3) 10.40a.m. 4) None cS OE IS SS ete te 10. The bells of a church ring for every § hours, The last bell already rang 95 minutes ago and the next bell will ring at 6.30 p.m. At what time did the priest give this information to his disciple? 1) 3.45 pm 2) 4.55 p.m. 3) 3.05 p.m. 4)4.05p.m. 11, Mr. Rama Rao goes to Hospital every day for checkup. On monday he goes at 6.30 p.m. Every day he goes 10 minutes late than the previous day. On Friday at what time will he moot the doctor? 1)7pm. 2) 7.05 p.m. 3)7.10p.m. 4) 7.15 p.m 12. Ramana had left his house at 15 minutes to 8 in the morning, reached Vijay's house in 25 minutes. They had finished their breakfast in another 10 minutes and left for their office which took 40 minutes to reach there. At what time did Vijay reach his office? 1) 8.55 a.m. 2)8.20 a.m. 3)9 am 4)9.15a.m. 13. After reaching the place of meeting on thursday 20 minutes before 9.30 a.m. Arun found himself 30 minutes earlier than the man who was late by 45 minutes. What was the scheduled time of the meeting? 1)8.45 a.m 2)9.10a.m. 3)8.55a.m. 4) 9.35a.m. 14. Javed left college to reach the bus stop 10 minutes earlier than his usual time. It takes 15 minutes to reach the stop. He reached the stop at 5.50 p.m. Atwhat time does he usually leave college for the bus stop? 1)5.30p.m. 2)5.45 p.m. 3)5.15p.m. 4) None 15. Which two months in a non - leap year have the same calander? 1) March, May 2) June, Ostober 3) April, July 4) Sept., November 16. Ifthe first day of a non-leap year was saturday then what will be the day on 25th Decomber of that year?, 1) Friday 2) Sunday 3) Saturday 4) Monday 17. Priya reached a place on Friday she came to know that she was three days earlier than the scheduled day. If she had reached there on the following Sunday, how many days late/early would she have been? 1) One day earlier 2) One day late 3) Two days late 4) Two days earlier 18. Which one of the given responses would be a meaningful order of the following ? 1) Probation 2) Interview 3) Selection 4) Appointment 5) Advertisement 6) Application 1)5,6,3,2,4,1 2)5,6,4,2,5,1 4)5,6,2,3,.4,1 Creative Learnina inétitute for Comnrchencive Knowledae - CLICK Amonkey climbs 30 feet at the begining of each hour and rests for a while when it slips back 20 1 feet before. It again starts climbing in the begining of the next hour. If it begins the process at 8.00 a.m., at what time will it touch a flag at 120 feet for the first time from the ground ? 1)3 p.m. 2)5p.m. 3)6p.m. 4) None It the ‘May day’ falls On Saturclay, in a leap year then which day will be the Birthday of Mahatma Gandhiji in the samo year ? 4) Saturday 2) Sunday 3) Monday 4) Tuesday 18 2)2 3/2 «4/4 5)36}4 73 BA QT. 1093 113 12)3 13)3 14)2'15)3_16)2_-17)1-18)1-19)2-2O)1 tHe Creative Learning Institute for Comprehensive Knowledge : CLICK cid The face or dial of a clock is a circle whose circumference is divided into 60 equal parts, called minute space. + The clock has two hands - the short hand and the long hand. a Short hand indicates time in hours and the long hand indicates time in minutes. ° The hour hand moves around the whole circumference of the clock in 12 hours # The minute.hand moves around the whole circumference of the clock in 1 hour e Thus, the minute hand is twelve times faster than the hour hand. + The minutes hand passes over the 60 minutes spaces while hour hand goes over the 5 minute spaces. That is, in GO minutes, the minute hand gains 55 minutes over the hours hand or 55 / 60 minutes spaces in 1 minute. 6° + 1 minute ae inute space = “05 none minute, the minute hand moves 6° 360. 12x60 ° In one minute, the hour hand moves =. o *# Thus, in one minute the minute hand gains 5 over the hour hand. # The hands coincide to make an anige 0° once for every one hour in twelve hours, the hands coincide 11 times (between-11 and 1 O'clock there is a common position at 12 O' clock). + The hands lie on the straight line to. make an angle of 180° once for every 1 hour. In twelve hours, the hands lie on the straight line for 11 times (between 5 and 7 O'clock there is a common position at 6 O' clock). © The hands of clock are at right angles (or) 90° twice in every hour but in 12 hours they are at right angles 22 times only since there are two common position in every 12:hours. (They are 3. a.m., 9a.m.,3 mand 9 p.m.) oe Creative Learning Institute for Comprehensive Knowledge: CLICK UnetanohtamniiceAiaoc i eeemmccnncamemn "” Raabe Se or Te oN a ated EXERCISE Atwhat time between 4io clock and 5'9 clock do the hands of a clock point in opposite directions? “see S é s 1) 4 past eT, min. 2) 4 past 50 min. et 6 3) 4 past oy mih. 4) 4past By min. At what time betiveen 3'o clock and 4'o clock do the hands of a clock are at right angles 7 ; s 1) 8 past a min. 2) 3 past 30 min. Bo : 3) 3 past 3277 min. 4) 3 past 40 min. At what time between 7’o clock and 8'o clock do.the hands of a clock coincide ? )7 past 35 mk 2) 7 past 38 mi 1) 7 past 36> min. ) 7 past 3855 min. 8 3) 7 past a min. " 4) 7 past 35 min. At what time between 2'o clock and 3'o\¢lock do the hands of a watch be together ? 7 % 10 4) 2 past 10 min. 2)2past 11> min. 10 2 9) Zpast 10> min. 4)2 past 12" min ‘At what time between 5.30 and 6'0 clock do the hands of a clock be at right angled ? 6 5 2 1) 5 past 42> min. 2)5 past 40 min, e : Pee 3) 5 past 42— min. 4) Spast 43 min. ” Atwhat time belween 5'0 clock and 6'0 clock do the hands of a clock be in the sume ‘Straight line but nol opposite to each other ? 3 1) 5 past 25 min. 2)S past 275 min. z 2 3) 5 past a min, 4) 5 past UT min. 10, Leh held me At what time between 3'o clock and 4’o clock do the hands be 2 minutes spaces apart ? 1) az va 16 thie: 2) a rane 183 it 3) ut min., 185. in ayia reli 186 min At what lime between 4'o clock and 5'o clock do the hands of a clock lie 3 minutes apart ? 1)17F in, 25:2 min, 2) ss ade ast ae 3) 162 soins; we nie 4) sz mites ne nai At what time between 9'0 clock and 10’o clock do the hands'of a Clock lie 4 minute spaces apart? 2 ie 7 5 45— min., 55— 43— min., 56— mi 1) 4555 min., 8555 min, 2) 43 min., 36 min. 8 5 2 8 44— min., 53 mir 43— min., 5S2— 9) 447 min., 9355 min. 4) A355 min., 5255 min. The mirror image of a clock shows 3.40. What Is the correct time in the watch ? 1)7.40 2) 8.25 - 9)9.20 4) 8.20 The time at Hyderabad is 6 hours 40 minutes behind the time at London. If a watch read 2.20 p.m.on October 19th at London, what is the corresponding time at Hyderabad? 1)9.10a.m. Z 2)11.40a.m. 3) 7.40 a.m. 4) 6.40 a.m, At what time between 6 and 6:30 will the hands of a clock be inclined at an angle of 70° 7 1) 10 min. past 6'o clock 2) 20 min. past 6'o clock 3) 8 min. past 6'o clock 4) 12 min. past 6'0 clock For how many times, the hands of a clock be at right angles in 12 hours ? 1)44times 2)22 times 3) 12 times 4) 11 times The minute hand of a clock, overtakes the hour hand at an interval of 64 minutes of a correct clock. How much does the clock gain or lose in a day ? 8 2 8 8 30— mi 38> mit 32— min. 32= min. 1) 30 min. 2) 385 min 3) 3255 min. 4) 32.5 min Creative Learning Institute for Comprehensive Knowledge: CLICK The hands of a correct clock coincide after every 6 5 1) 59 min. 2) 64 — min. 3) er min. 4) 60 min. i. Atwhat time between 8 and 9'o clock will the hands of a clock be opposite to each other 7 9 lo 1) 0, min. past 8'0 clock 2) er min. past 8'0 clock lo. 9 3) as min. past 8'0 clock 4) oa min. past 8'0 clock ‘The mirror image of a clack shows 3.20 hrs. Whatis the actual time ? 1) 7.40 2)9.40 3) 8.40 4)6.40 \. Acclock strikes once at 1'o clock, twice at 2’ clock, thrice at 3'0 clock and so on. How many times will it strike ina day ? 1)240 2)290 3)300 4) 920 A clock strikes 4 in 12 seconds. In how many seconds will it strike 8 2 1) 18 2)21 3) 24 4)27 |. A.clock strikes 5 in 8 seconds, in how many seconds will it strike 10? 4) 12 214 3) 18 4)16 yt 2/3 -9)2 43 54 W)2 7/4 -B)2_ sD 10)4 11)3 12)2 -13)414)3_—'15)3.'16)2 173 -18)3-—-19)3 2O)4 toe REECE Oe ARO" RIN ag eed CALENDARS Duration of period in terms of days ; (Number of days in a leap year is given in brackets) Period Number of days February 28(29) 1* quarter : Jan 1 to March 31 90(91) 2° quarter : April 1 to June 30 ot 3% quarter ; July 1 to sep 30 92 4" quarter : Oct 1 to Dec 31 92 Jan 1 to Dec 31 365 (366) A leap year is a multiple of 4, However, the century years should be divisible by 400. (Thus 400, 800, 1200, 1600, 2000 etc. are leap years). The points to remember : i) w) 1) (iv) ) (vi) (vi) i. Tet Duration of period in terms of hours minutes and seconds. Detarmination of the day. A day repeats itself after’7, 14, 21 —-- 364 days (including the present day) In an ordinary year there are 365 days i.e. 52 weeks + 1 days. Therofore an ordinary year contains 1 odd day. Therefore for a non-leap year, the weekday of a specific date will increase by one in the next year. In a leap year there are 366 days i.e. 52 weeks + 2days. Therefore it contains 2 odd days. For each leap year the day of the week advances by two for the next year. 100 years = 76 ordinary years + 24 leap years ~ 76 odd days +24 x2 odd days = 124 odd days = 17 weeks + 50 odd days :, 100 years contain 'S“odd days ~. 200 years contain '3' odd days ~. 800 years contain '1' odd day ~., 400 years contain ‘0’ odd day First January 1 A.D. was Monday, Therefore, we must count days from Sunday i.e. Sunday for 0 odd day, Monday for 1 odd day, Tuesday for 2 odd days, Wednesday 3 odd days, Thursday 4 odd days, Friday 5 odd days and Saturday for 6 odd days. Last day of a century cannot be a Tuesday, Thursday (or) Saturday. be Monday, Tuesday, Thursday (or) Saturday. Creative Learning Institute for Comprehensive Knowledge: CLICK. EXERCISE January 1°" in 2007:is Moriday. What day of the week will itbe on January 1, 2008 ? 1) Monday 2) Tuesday 3) Wednesday 4) Sunday August 5" in 2007 falls on Wednesday. What day of the week was it in 2005 ? 1) Tuesday 2) Thursday 3) Monday 4) Friday Itwas monday on 4"" April 1988. What wasthe day on 3 November 1988 7 1) Tuesday 2)Wednésday 3) Thursday 4) Friday Today is Monday. What is the day of week after 68 days 7 1) Sunday 2) Saturday 3) Friday 4) Tuesday Gandhiji died on 30" January 1948. The day of the week was 1) Thursday 2) Friday 3) Monday 4) Saturday YSR became the CM of Andhra Pradesh on 13" May 2004. What was the day of the week ? 1) Monday 2) Tuesday 3) Wednesday _ 4) Thursday If 14-1-2008 falls on Monday, what will be the day of the week on 18-1-2009 ? 1) Tuesday 2) Monday 3) Friday 4) Sunday if 1% of a month falls on three days after Monday, what will be the day of the week on 18" of that month ? 1) Monday ; 2) Saturday 3) Friday 4) Sunday Any date in March is always on the same day of the week as the corresponding date in the month of 2 1) April 2 +2) November 3) October A) July Any date in April is always on the same-day of the week as the corresponding date in the month of 1) March 2) August 8) July 4) June How many days are there from 3 Jan 1995 to 16" March 1995 ? 72 2)73 3) 74 4)75 Today it is Saturday. What is the day of after 120 days ? 1) Saturday 2) Sunday 3) Monday 4) Friday If 4-1-2004 was Sunday, what was the day of the week on 6-1-2005 ? 1) Tuesday 2) Thursday 3) Friday 4) Saturday Reet Sit atin athe nner ames oh Mai atmninee antic tla ines mactAicatoR ictal 14. If 3 of a month falls on the day preceding Saturday on what date will fourth Thursday fall ? 1)24 2)19 3) 23 4) 24 15. Today is 2" February. The day of the wook is Thursday. Last year was a leap year. Tho day of the week on this date:three years ago was 1) Tuesday 2) Friday 3) Sunday 4) Monday 16. Itwas Sunday on sit Jan 1993. What would be the day on a 14 March 1993 ? 1) Monday 2) Saturday 3) Sunday 4) Friday 17. Ifthe third day of the year 2000 is a Monday, then how many Mondays were there in that year? 1) 50 2)51 3) 52 4)53 18. On what day of the week was 1% January 1975 ? 1) Tuesday 2) Wednesday 3) Thursday 4) Sunday 19. The year next to 1987, that has the same calendar as that of 1987 is 1) 1988 2) 1997 3)1996 - 4) 1998 20. The year next to 1995, thathas the same calendar as thal of 1995 is 4) 2001 2) 2006 3) 2007 4)2010 12 23 33 42 51 64 7)4 84 92 10)3 11)2 12)3 13)2 14)3 -15)3-16)4 -17)3. 182-1994 -20)2 bbb Creative Learning Institute for Comprehens! PREVIOUS ICET QUESTIONS . Series 80-84 . Classification (odd thing out) 85 - 87 . Coding and Decoding 88 - Of . Date, Time & Arrangement Problems 92 - 94 2, 8, 18, 82, . ony 72, 98, 1)48 2) 52 4, 13, 28, 49, 109, 148 4)76 2)70 0010,, 00114, ... O11, 1011, 1) 1010, 2) 1000, 0,7, 26, , 215, 342 1)126 2) 124 ACEG, IKMO, YACE 1)QSUW 2) PRTV By 1G, BE spss cesennsrecny 4OQ, 221 ye 2)93 5, 10, 26, 122, 170 4) 48 2)50 ABC, DEF, WK, .. sos YZA 4) MNO 2)LMN 6:95:: 77: 1) 135 2)221 Bs 18: : 125.4 = 1)426 “2)198 “ AEI, OGK, . GKO, IMG 1). EIM 2) FUN AH AL, HE Ys 2)T 216, 343, ... a 72D 1)470 2)512 BDF, DHL, HPX, wstanesonny FLR 4) JFV 2)PGV 05-01-1996, 27-01-1996, 18-02-1996... 1) 08-03-1996 2) 28-02-1996 83 3) 48 a)72 9) 0101, 3) 125 a)osuv ‘379, 3)49 3)NOP. 3) 225, 3) 625 3) DHL 3)Vv 3) 570 3) PFV +», 02-04-1996 8) 12-08-1998 4)50 474 4)0110, 4) 127 4) QSvx 477 4)53. 4)POR 4321 4)526 4) BJM au 4) 626 4) PFU 4) 11-03-1996 Eldred CRO SU) °c ch eee ee ead 16. V21, W3G, X5E, Y7C .. 1.2115 2.798 17. ACZY, CEYX, EGXW,. 1.G1ZY 2. GIWV 18. ZOA,Y38C,....... -.W15G, V24l 1. XOE 2. X8F 19. DFIK, GILN, JLOQ,.. 1. MORT 2. MRPO 20. ZYK, WVU, ,QPO, NML, Kul 4. SRT 2.TSR 21. 3,5,7, 11, 13, 17, 19,.. 4.24 2.25 22. 5,7, 12, 19,3 1.36 2.40 28. 15,32, 66,... us 1.136 2.134 24. (0, 1), (1,2), (2,5), (3, 10).. 1.(4, 17) 2. (4, 18) | 25. 2,6, 12, 20, 30, 1.38 240 . 26. EJO, DHL, CFI, BD 4,ABC 2. ACE 27. HLTX, JMSV, LNRT, NOOR... 1. PPQR 2. RPPP 28. 29. 3.Z9A 8. GIVU 3. X8E 3.MORP 3.TRS 3.23 3.140 3, (4, 19) 3.42 3. ABD 3. QPPQ 4.Z11A 4, GIWU 4.X9F 4. MPRO 4.RST 4 27 4.50 & 138 4. (4,21) 4.44 4, ABE 4. PPPP 4.172 4.87 ed 0. 1/2,8/, 2716,... 1.81/7 A. 0010, 0011, 0101, 1.0001 B28, 8 ac 1.6 2.11 8. AZBY, CXDW, . 1, EXUC 4. 9,19, 40,...... 1.70 eke ee + 128/11, 21613 1. 125 2.216 7. 11409, 12 1/2, 14.2/7, 182/83, 1.1828 2.182/7 8 165, 196, ..... 1,235 9. 09:25:49 1.64 0. 13, 29, .. 1.49 1. 583: 283 :: 488: .. Pons 1.387 2.378 2. CAT, FDW, IGZ........ 1,.UIC 3. 82:122:: earn 1.154 2,145 4. GHD, JEG, MB4,. oi 1,Q2M 2.PYM 5. D4V, FET, . 1. K7P. 3.81/9 3.1010 3.7 3. EVRU 3.69 3.23 3,124 3.1746 3.275 3.36 3.478 3. WB 3.185 3.PZM 4.163 4,20 4,255 4.68 4.72 4.368 4.LJC 4.170 4, PYN. Dade b ol 46. St, 55. . 2,6, 12, 20, . 0.5, 0.55, 0.65, 0. . 7/11, 13/17, 19/28, .... 2,9, 28... 1.47 3,17, 85,129, .. cst 1.249 = 433 2.250 1.26 1.0.9 2.0.82 37/41, 43/47 1.29/81 2.31/29 6, 26, 126, 626, .. .., 15626 1.4126 2.2126 . (4, 15), (6, 35), (8, 63), nssreecsrenes (12, 143) 1.(10, 93) 2. (10, 99) . ACE. GIL, JLO, MOR 1.8DG 2. CEH . ABYZ, CDWX, .... .. GHST 1. EFVU 2. EFUN 2A4,3E5, 416, ..... soy 6U8 1.5M7 2.5N7 2002 56. 87. 58. Se, STOP, STPO, SOTP, SOPT, SPTO.... 1. SPOT 2. STPO BE: HK :: 1.MP 2.NP 86 3.63 9,252 3.30 3.21/29 3.3126 3, (10, 98) (3. DFI 3. FEUV 3.507 3. SOPT 3.NQ 3.57 PRI ld 4.65 4.251 4,32 4 0.95 4.49/21 4.7126 4. (10, 96) 4.EGJ 4. FEVU 4.5P7 4, SOTP 4.MQ 4.75 i i mala 3.169 3.ED 3. 48 3.1233 3.59 3.7518 9.4185 3. ABXYCD. 30. 6, 18,54,...... 4.108 31. BD, CE, DF... 1.BA 2.EB 52. 6,14, 24, 36,....... 1.40 2.44 ig. a , 2842, 3453, 4564, 5675 1.1231 2.1232 34. 136, 100, 76, ... sn», 80, 60, 45 1.53 2.55 35. 15, 8/9, 27/18, . 125/21, 216/25 1.35/16 2.64/17 36. 82: 122: $226 1.145 2.170 37. ANXYBO, CPXYDQ, ERXYFS...... 1. GTXYHU 2. DOXYPS 1)4 2)1 33 64 2 ae 8)4 41)4 12)4 13)2 164 172 18)1 21)3 22)4 23)2 26) 1 27)4 28)1 31)2 932) 4 33)2 36)2 37)4 38) 1 41)2 42)2 43) 4 46) 4 47)4 48) 3 51)3 52)2 53)3 56) 1 57)3 58) 2 61)4 62)4 63)1 66)2 67)1 4,381 4.EG 4.1234 4.60 4.81/19 4.210 4. CEXYFG 5)1 10)3 15)4 20) 2 25) 3 30) 2 35) 2 40)3 45)3 50) 1 55) 1 60) 2 65) 2 CLASSIFICATION fy 23 a5 a7 2 1946 2)58 3)69 4)74 8-135 277 a9 4) 63 4, 1)180 2) 207 3) 240 4) 147 5. 1)29 2)47 3)5% 4)67 6 1)95 2)91 9)92 4)97 7 1B 2)0 aI 4)P 8 1)BAT 2) CAT DOT 4) PAT 9 1)5 2)50 3122 4) 169 40. 1)961 2) 531 8) 169 4841 2006 ’ 11. 1, August 2. July 3. May 4. June 12. 1 Fish 2.Ftog 3. Crocodile 4.Turtle 13. 1.1004 2.1011 3.1101 4.414 44, 1. UNR 2.0MS 2. DLT - 4. AKU 15, 1. AX2T 2.XZTA 3, ZTAX 4. TAZX 16. 1.67 2.57 3.47 4.37 17. 1.15n4~ | 2.2/8 3.23/24 4.3195 18, 1, EYE 2. TONGUE 3. EAR 4. NOSE 19. 1.216 2.343 3.516 4.729 20. 1.14 2414 “3.4114 4.411111 2005 21, 1.65 2.126 3.217 4,343 22, 1.1549 2.14/43 3.37 4.215 23. 1.345 2.143 3.567 4.789 24, 1.169 -2.961 3.131 4.625 25, 1.DELM 2. BDI 3. GHRS 4. POAB 26. 1.BFH 2.MQS 3. GUL 4, NRT 27. 1. PLANET 2. SATELLITE 3. SKY 4. STAR 28. 1.BAT 2. EAT 3. FAT 4. POT Creative Learning Institute 15 Comprehensive Knowledge : cuck © 23-29 a 3. 1, ERP 125 87 23 124 1G - PRK + AKU 0.4 1.101 1.191 Ret ket Bet 4 5 3. a. tat 3 2 1. 1.6622 2, 1.129 3. 1.CBA 4. 1, ACE 5, 1.0A 5. 1. X¥Z 74 2.957 2.64 23-5 2.45 2, 384 2.63 2.10 2.27 2. ABC 2. REP 26S 2,091 2.27 2.77 2.33 2.142 2.VE 2.1RK 2. CMW 2.0.04 2.170 2.2002 2. 8844 2.127 2.DCB 2. FHS 2.1U 2. PQR 2.VS 3.894. 3.48 3.13-17 3.55 3.843, 3.36 3.29 3.541 3. XYZ 3. PRS. 3.DQ 3..D25Y 3.32 3.67 3.43 3.241 3.XB 3, EST 3, ENX 3,0.404 3.196 3, 3003 3, 9933 3.125 3. EDC. 3. SUV 3. EB 3. LMN 4.876 4.78 4.7-19 4.65 4, 834 4.43 4,66 4.716 4. PAR 4, PER 4.uW 4.Y9 4.34 4.57 4.53 4.214 4.Z0 4, ALN 4, DNX 4, 0.0004 4.226 4, 400004. 4.3311 4,123 4. FEP 4.RTV 4.Al 4.TSR Creative Learning Institute for Comprehensive Knowledge : CLICK 14 6)3 11)4 16)2 21)4 26) 3 31)4 36) 4 41)2 46) 1 51)2 56)4 21 D3 12)4 174 22)1 27)3 32)2 37)3 42)3 47)4 62)2 57)1 34 83 13)4 18)2 28)2 28) 2 33)3 98) 3 43)2 48)3 53) 4 toe 42 94 14)3 19)2 24) 3 29) 1 34)4 99) 2 44) 3 49) 1 54) 3 ~6)3 10)2 15)4 20) 2 25)2 30) 2 35)1 40)1 45) 3 50)3 55)3 fate icy PREVIOUS ICET QUESTIONS CODING - DECODING Inacertain code the words HONEST, EXAMINATION, BOY, RIGHT and WRONG are respectively written as UBARFG, RKNZVANGVBA, OBL, EVTUG and JEBAT. Find the process of coding and answer the cvestions, ‘The code tor FIGHT is A) TVTUG 2)SVTUH 3}SWTUG 4) SVTUG The code for ICET is 1)UPRG 2) PRG 3) VORG 4)VPSG The code for ARMY is 4)NEZM 2) MEZL 3) NEZL 4) NFZL The code for SOLDIER Is 4) FBYQVRE 2) FBZQVRE 3) FBYQVSE 4) FBYQURE The code for SEVEN is 4) FRIRZ 2) FRJRA 3) FRIRB 4) FRIRA Which word is coded as MINISTER ? 1)ZVAVFGRE * 2) ZVAVFHRE 3) ZVAVFGSE 4) ZVAVFGRF Which word is coded as KING ? 4) XVAS 2) XVAT 3) XVAU. 4) XUAT Which word is coded as HISTORY ? 1) UVFGBEM 2) UVFGBEL 3) UVFGAEL 4) UVFGBEK Which word is coded as PLAN 7 AVOYNZ 2) CYNB 3) CYNA 4)CYND |. Which wordis coded as QUESTION ? 1) DHRFGVBA 2) DHRFGUBA 3) DHRFGVCA 4)DHRFGVDA Inacode, the r letter is shifted to (27 - 2r)" letter for r=1, 2,......,13, the urteenth letter is shifted to 26" letter and, for r= 15, 1 .26, the r® latter is shifted to (2r - 28)" tter. For decoding the inverse process of the above is followed. Using this coding’and decoding, iswer the questions. |. Which word is coded as ITALY? AAXTMA 2. 1XMTY 3. 1XMTA 4, IMXTY 2. Which letter is coded as Y?. 1A 2N 3.B 4M 3. What is the code letter for P? REASONING 14, 15. What is the code word for SENDER? 1. JQSZQH 2. JRSZRH What is the code word for POTA? 1. DLBY 2.DBYJ — | Which word is decoded as LMVIXMT? 1. CAPSTAN 2. CAPITAL . What is the code for IUPILM? 1. INDIAN 2. INDIRA What is the code word for MAGNET? 4. ANMQZL 2. AYZMQL |. What is the.code for HIXVTA? 1. KITKAT 2. KITPLY ). Which word is decoded as XIDLF? 1. THINK 2. TIGER Directions:(@.No, 21-30): In acode TANK is written as SZOL and FRIEND is written as EQHFOE. Find the process of coding and answer the questions 21. 22. 23. 26. The code for RING is 1. QHOG 2. QHOH The Code for FROG is 1. EQPH 2. GSPH The code for ZENITH is), 1. YDMJUI 2. ADMJUI .. The code for PARADE is 1. OZOZEF 2. OZPBEG The Code for PIPE is 41. QJOD 2. OHOD . Which word is coded as BATS? 1,CBTR 2.CZSR . Which word is coded as COURSE? 1. DPVPRD. 2, DPUPRD. 3. JPZSPH 3. DBLY 3. CAPTAIN 3. INDUCE 3. AYMZQL. 3. KINLEY 3, TISCO 3. QHMF 3. EQNF 3. YFMJUL 3. OZOBEF 3. OHOF 3. CBST 3. DPVQRD 4. JQZSQH 4. DLYB 4. CAPTION 4. INDICA. 4. AYQMZL “4. KINDLE 4. THICK 4. PHOH 4. GSNF 3. ADMJUG 4, OZOBFE 4. QJOF 4.CBSR 4, DPVORD Cried Paoli eam cued Which word is coded as DEMAND? 1. EFNBMG 2. EFNZMC 3. DENZMG 4. EFNZLC Which word is coded as NUMBER? 1. OVNBES 2. OVOADQ 3. OVNZDQ 4, OVNADQ Which word is coded as RATE? 1. SBUF 2. SBSD 3, SBRD 4. SZSD. re tions:(Q.No. 31-40): Ina Code, each letter in English alphabet is shifted forward to five places sical, that is Amp F, Ba G, , UZ, Vea A, Wa B, Z=>E. The re- *3@ ofthis process (s used for decoding, Based on this coding and decoding processes answer the estions. . The code for MATHS is 1 REMYZ 2.RFYMX 3.RFYMS 4.REXMY + The code for BUSINEES is 1.GZXXNTJIXX 2GZXXNSKXX 3.GZKXNSIJXX 4.GZVYNTJVY . What is the code wofd for ICET? 1.NHKZ 2NHKY B3.NHJX 4.NHJY - What is the code word for EXAM? 4, JCFS 2. JCFR. 3. JCFT 4, JCFQ . What Is the code of PRIMARY? 1.UWNRFWD 2UWNRFXD , 3.UWMRFWD 4.UWNSFWD . Whal word is coded as INDIA? 1.DJYDV 2.007 DU 3.DIYDV- 4.DJYDW . Which word is coded as KARGIL? : 1.FVMBEG 2FVMBDJ 3.F VMBDH 4.FVMBDG . Which word is coded as NEXT? V1ZSP 21280 S1ZSN 41ZTO . Which word is coded as BANGALORE? LWVIBVGJMZ 2XJIQZIZN 3.XJTQYIYM 4.XJIQZIZM Which word is coded as CONVENER? A XJIQYIZN 2.XJ!1QZIZN S.XJIZYIYM 4. NONE REASONING 1)4 62 11)3 16)2 212 26)4 - 31)2 36) 3 22 12 12)4 174 22)1 27)3 32)3 37)4 3)3 8)2 19)2 18)3 23)1 28) 2 33) 4 38) 2 +e 41 93 14)4 19)2 24) 3 29)4 34) 2 39) 1 5)4 10) 1 15)3 20) 3 26) 3 30) 2 35) 1 40) 4 DATE, TIME & ARRANGEMENT PROBLEMS The time on the clock is 3.00 p.m. if the hours hand is pointing towards West, then the direction of the minutes hand is 1) North 2) South 8) South-West 4)West If the first day of June is a Saturday, then the date on which the last Saturday of that month falls is 1)22 2) 28 3)29 4)30 IF 9" March of 1996 is a Saturday, then the 9" March of 1996 is a . 1) Wednesday 2) Tuesday 3) Sunday 4) Monday The ratio al the present ages of a tather and his son is 2: 1. If the ratio 10 years ago is 5 : 2, then the present age o! the son is 1)30 2)25 324 4)32 Ais the father of B and C;E is the mother of C end Dis the wife of F. If F is the brother of E, then how D is related to B? 1) Maternal grand-mother 2)Maternal aunt 3) Paternal grand-mother 4) Paternal aunt If 8 March falls three days after Sunday, on what day will the last day of that month fall? 1. Wednesday 2. Thursday 3.Friday 4, Saturday In-a clock, What is the angle between the two hands al 5 hours 10 minutes? 1.60° 2. 95° 3.90° 4.100° The last day of February 2006 Was a? 1, Monday 2. Tuesday 3. Wednesday 4, Thursday In the array 48392874362754869364, the number of instances Where an even number is followed by two odd numbers is? wd ae 33 4.4 Haw many integers from 1 to 100 exist such that each is divisible by 5 and also has 5 as a digit? 1.10 2.11 3.12 4.20 Lis) . The time on the watch is 9-15 and the hour hand points towardas west. The direction of the minutes hand is? 2. South ee 12. Ina row of Six persons D and C are immediate neighbours of E. Bis a neighbour of A only. A is the fourth from F. Who are on the two end points? : 1.78 2.40 3.B,D 4A 18. While climbing a 40 feet tall pole, a monkey ascends ¢ feet in a single jump, but slips down 2 feet immediately. How many jumps does it require to reach the top of the pole? 1.10 mat 3.20 4.19 14. Five books are lying in a pile. E is lying on A and C is lying under B. A is lying above B and D is lying under C. Which book is lying at the bottom? 1A 26 B.D 4.8 15. A siarts from his home and goes two kilometers straight. Then he turns towards his right and goes one kilometer. He turns again towards his right and goes one kilometer. If he is North- Wost from his house, then in which direction did he go in the beginning? 1. East 2. West 3.South’ 4, North : 16. A clock is set right at 5 a.m. The clock loses 16 min. in 24 hours. What will be the true time when the clock indicates 10 p.m. on the fourth day? 1.10:30 Pm 2.11pm 3.11 :30 pm 4.10: 45pm 17. On July 2, 1985, it was Wednesday, The day of the week on July 2, 1984 was? 1.Monday _ 2. Tuesday 3. Wednesday 4, Saturday 48. The mirror reflection of clock shows 02:0 hrs. What is the actual time? 4.09:40 hrs. 2.08: 30hrs. “3.10: 30hrs. 4.09: 80hrs. 19. If the last day of March is Wednesday, the day of which the month starts is: 1. Monday 2. Tuesday 3. Thursday 4. Friday 20. A,B,C, D, E & Fara soatad in a circle facing the centre. Dis between F and B, A is second to the left of D and second to the right of E. Who is facing A? 1.0 2.ForB 3.CorD 4.E 21. Ais 40 meters South-West of B and C is 40 meters South-East of B. Then C is in which —_ direction of A? 1 East 2, West 3.South 4. North 22. Aand Bare children of C. Bis the mother of D and E is maternal grandmother of D. What is the relation of E to C? 1. Husband 2. Sister 3. Wife 4. Brother 1)2 6)3 11)3 16) 2 21)1 2)3 7)3 12) 4 17)1 22)3 3)4 4)1 8)2 9)3 13) 4 14)3 18) 4 19) 1 5)2 10)2 15)2 20)2

You might also like